ASIPP Diagnosis and Therapy Questions Flashcards
- Proposed mechanisms of action for spinal cord
stimulation include all of the following except:
A. Segmental antidromic inhibition of A-beta afferents
B. Blocking of transmission in the spinothalamic tract
C. Supraspinal pain inhibition
D. Activation of central inhibition of sympathetic efferent
neurons
E. Antidromic activation of C nociceptive afferents
- Answer: E
Explanation:
Reference:
Krames, Interventional Pain Management,Second Edition;
Chapter 53 Mechanisms of Action of Spinal Cord
Stimulation
A. Segmental activation of large A-beta fi bers within the
dorsal columns which antidromically inhibit reception of
small fi ber nociceptive information at the substantia
gelatinosa of the dorsal horn.
This was Melzack and Wall’s original hypothesis and is
consistent with a classic “gate control” theory of spinal
cord stimulation.
B. Segmental blockade of neurotransmission in the
spinothalamic tract.
This theory is supported by studies that show there is
inhibition of pain transmission locally within the cord
during spinal cord stimulation.
C. Spinal cord stimulation produces changes in
supraspinal neurons that either modulate supraspinal
pain transmission or trigger supraspinal descending
inhibition of the dorsal horn.
D. Activation of central inhibition of sympathetic efferent
neurons could affect pain processing. The consistent effect
of vasodilation supports a sympathetic inhibition effect of
spinal cord stimulation.
E. Release of putative neurotransmitters and/or
neuromodulators. This theory is based on the observation
that pain relief often outlasts the duration of stimulation
for minutes, hours and sometimes days.
Source: Schultz D, Board Review 2004
740. For brachial plexus avulsion pain, the long-term outcomes of DREZ lesioning are approximately: A. 60-65% B. 10-15% C. 1-2% D. 40-50% E. 15-25%
- Answer: A
Explanation:
(Raj, Practical Mgmt of Pain, 3rd Ed., page 802)
Long term relief of brachial plexus avulsion pain with
DREZ lesioning is 60-65% at 3-7 years. Phantom and
stump pain success is about 50-60%. Spinal cord injury
pain is usually not responsive, except for end zone pain
(segmental) occurring just below the level of injury. 70-
75% of patients report successful relief with end zone pain.
Source: Schultz D, Board Review 2004
- The anterior spinothalamic tract:
A. Is the primary target of a cordotomy
B. Primarily conveys proprioceptive afferent fi bers
C. Primarily conveys small fi ber afferents.
D. Conveys light touch.
E. Conveys temperature sensation
- Answer: D
Source: Feler C, Board Review 2005
- Which of the following carries the lowest risk of
complications?
A. Microvascular decompression
B. Subtemporal sensory rhizotomy
C. Selective trigeminal rhizotomy
D. Open trigeminal (nucleus caudalis) tractotomy
E. Stereotactic trigeminal tractotomy
- Answer: A
Explanation:
(Raj, Pain Review 2nd Ed., page 311, Raj, Practical Pain
Mgmt, 3rd Ed. Page
798, Bonica 3rd ed. Page 2042-2043)
Microvascular decompression, initially developed by
Janetta, is a non-destructive and potentially curative
operation for trigeminal neuralgia. A pulsating aberrant
vessel loop, e.g., superior cerebellar artery (V3),
trigeminal vein (V2), or anterior inferior cerebellar artery
(V1), is felt to be the cause. An interposition felt of Tefl on
or polyvinyl sponge is placed between the vessel loop and
the trigeminal nerve. Recall, V1+V2+V3 –> TG –>
Trigeminal Nerve –> Brainstem. Although, complications
such as cerebellar or brainstem strokes, CSF leaks,
meningitis…complication rates are
743. Dextrose is added to lidocaine, during a spinal anesthetic. Which position would most likely result in anesthesia of the sacral dermatomes? A. Prone B. Side-lying C. Jack Knife D. Sitting upright E. Trendelenburg
- Answer: D
Explanation:
(Raj, Practical Management of Pain, 3rd Ed., page 632,
635)
Baricity of a local anesthetic is described as the density of a
local anesthetic solution divided by the density of CSF. The
density of CSF is 1.001 to 1.005 at 37°C. Local anesthetic
solutions are characterized relative to CSF as hyperbaric,
hypobaric, or isobaric. Understanding this density
relationship allows the anesthesiologist to take advantage
of the characteristics of the local anesthetic or the position
of the patient to direct local anesthetic toward the
dermatomes to be anesthetized. A hyperbaric solution has
a higher specifi c gravity than CSF, so that it moves to lowlying
parts of the subarachnoid.
Although prone may result in blockade of the sacral
dermatomes, the sitting position would do this most
effectively, by having the bulk of the local anesthetic dose
go towards the sacral nerve roots.
Source: Shah RV, Board Review 2005
744. The most widely practiced percutaneous technique for relief of trigeminal neuralgia is: A. Glycerol rhizolysis B. Balloon-catheter decompression C. Microvascular decompression D. Radiofrequency thermocoagulation E. Radiosurgery
- Answer: D
Explanation:
(Raj, Pain Review 2nd Ed, page 311)
Radiofrequency thermocoagulation of the trigeminal
ganglion is the most widely practiced percutaneous
intervention. MVD and radiosurgery are not percutaneous
methods.
Source: Schultz D, Board Review 2004
745. What percentage of patients is affl icted by glossopharyngeal neuralgia compared to trigeminal neuralgia? A. 0.1-0.5% B. 1-2% C. 10-15% D. 30-40% E. 10-20%
- Answer: B
Explanation:
(Raj, Pain Review 2nd Ed., page 312)
Glossopharyngeal neuralgia affl icts 1-1.3% as many
individuals as trigeminal neuralgia. The lancinating,
paroxysmal qualities are similar but the pain is located at
the base of the tongue, throat, and deep in the ear. Triggers
include chewing and swallowing. GPN may rarely be
associated with syncope or bradycardia.
Source: Schultz D, Board Review 2004
746. Rotator cuff tear is diagnosed by: A. Plain radiographs B. MRI C. MR arthrography D. CT E. Sonography
- Answer: B
Explanation:
MRI provides the greatest imaging resolution and
complete evaluation in the setting of shoulder pain.
Although a rotator cuff tear may be diagnosed with
ultrasound or MR arthrography, diagnostic MRI remains
the best modality in MRI compatible patients
Source: Bieneman B, Board Review 2005
747. As a part of a psychological evaluation, a clinical interview includes all of the following EXCEPT: A. History B. Financial and legal information C. General medical status D. Psychosocial information E. Pain tolerance testing
- Answer: E
Source: Janata JW, Board Review 2005
- While performing a right lumbar sympathetic
radiofrequency lesioning, at L3, the patient complains
of pain in the right groin. What is the likely etiology of
this pain?
A. Lesioning of the ilioinguinal nerve
B. Psoas spasm
C. Lesioning of the iliohypogastric nerve
D. Lesioning of the genitofemoral nerve
E. Quadratus spasm
- Answer: D
Source: Day MR, Board Review 2005
- Which of the following is a true statement regarding a
thoracic sympathetic block?
A. Can only be performed with the patient in the prone
position.
B. Can be performed bilaterally at the same visit.
C. Pneumothorax is not a concern.
D. Blocks Kuntz’s fi bers
E. Not effective for treating thoracic visceral pain.
- Answer: D
Source: Day MR, Board Review 2005
- Which of the following factors infl uence the spread of
local anesthetic in the subarachnoid space the most?
A. baricity
B. barbotage
C. anesthetic dose
D. injection level
E. injection speed
- Answer: A
Explanation:
(Raj, Practical Mgmt of Pain 3rd Ed., page 635)
Factors affecting the spread of local anesthetic include (1)
baricity of the local anesthetic, (2) position of the patient
after injection, (3) level of injection, (4) speed of injection,
(5)dose and volume of the local anesthetic used, and (6) a
technique known as barbotage. Of these, the two with the
greatest infl uence are the baricity of the local anesthetic
and the position of the patient.
Source: Shah RV, Board Review 2005
751. The somatoform condition with the lowest incidence is: A. Factitious disorder B. Hypochondriasis C. Conversion disorder D. Somatization disorder E. Malingering
- Answer: C
Source: Janata JW, Board Review 2005
- The following is (are) true regarding Deep Brain
Stimulation (DBS):
A. Deep brain stimulation is an effective method for controlling
back pain.
B. It is FDA approved for painful conditions.
C. It is FDA approved for spasticity
D. It is FDA approved for some movement disorders
E. Two of the above
- Answer: D
Source: Feler C, Board Review 2005
- Which of the following is true?
A. “Radiculopathy” requires no neurologic defi cit
B. Any patient who has had prior lumbar spine surgery
who later presents with low back pain should be considered
to have FBSS.
C. A refl ex change alone is suffi cient to diagnose a
radiculopathy
D. Discogenic pain typically radiates into the affected dermatome
E. Two of the above
- Answer: C
Source: Feler C, Board Review 2005
- The following is not true:
A. MRI evidence of degenerative disc disease is necessary
for consideration of spinal instrumentation
B. A patient complaining of mechanical back pain who
has no evidence of instability is properly selected for
decompression of a nerve root without stabilization.
C. SCS is an FDA approved therapy for many indications
D. The L4 disc is usually the level of disease in a L5
radiculopathy
E. An absent ankle refl ex is indicative of a S1
radiculopathy
- Answer: A
Source: Feler C, Board Review 2005
755. Subarachnoid hemorrhage is best diagnosed by what test? A. Enhanced CT B. Unenhanced CT C. Enhanced MRI D. Unenhanced MRI E. Skull radiographs
755. Answer: B Explanation: Noncontrast CT brain is the appropriate exam for any acute neurologic abnormality Source: Bieneman B, Board Review 2005
756. Which of the following opioids should be avoided in a patient with renal disease? A. Meperidine B. Sufentanil C. Morphine D. Hydrocodone E. Hydromorphone
- Answer: A
Source: Day MR, Board Review 2005
757. Thalamic lesioning is usually used to treat: A. Shooting, allodynic pain B. Deafferentation pain C. Burning, dysesthetic pain D. Peripheral nociceptive pain E. Radicular pain
- Answer: A
Explanation:
(Raj, Pain Review 2nd Ed., page 309)
Thalamotomy is useful for intermittent, shooting,
hyperpathic or allodynic pain. Thalamotomy may not be
useful for burning, dysesthetic, central, or deafferentation
pain. Thalamotomy is not useful for peripheral nociceptive
pain. One of the most effective targets is the inferior
posteromedial thalamus.
Source: Schultz D, Board Review 2004
- A conversion disorder is:
A. Intentionally produced or feigned
B. Limited to pain
C. Suggestive of a neurological or general medical condition
D. Explainable by the effects of a substance.
E. Unrelated to functional impairment
- Answer: C
Source: Janata JW, Board Review 2005
759. Munchhausen syndrome is an type of: A. Hypochondriacal presentation B. Conversion disorder C. Somatization disorder D. Personality disorder E. Factitious disorder
- Answer: E
Source: Janata JW, Board Review 2005
- Which of the following is true about the stellate
ganglion?
A. Everybody has one
B. It is located at C6
C. It is formed by the fusion of the inferior cervical and fi rst
thoracic ganglion.
D. It is bordered anteriorly by the vertebral artery.
E. Blockade of the ganglion reliably causes a sympathectomy
of the ipsilateral upper extremity.
- Answer: C
Source: Day MR, Board Review 2005
761. Somatization disorder criteria include all of the following EXCEPT: A. A history of many physical complaints B. Onset after age 30 C. Four pain symptoms D. Two gastrointestinal symptoms E. One sexual symptom
- Answer: B
Source: Janata JW, Board Review 2005
- The presence of pain behavior in chronic pain
presentations:
A. is an indication of psychopathology
B. is abnormal in chronic pain populations
C. indicates that pain is “all in the patient’s head”
D. indicates the absence of true pathophysiology
E. is a normal adaptation to an abnormal set of circumstances
- Answer: E
Source: Janata JW, Board Review 2005
- With respect to cortical stimulation:
A. It is FDA approved for the treatment of atypical facial
pain.
B. There is suffi cient evidence based medicine to recommend
the procedure for patients with Anesthesia Dolorosa
of the face.
C. The procedure is easy to do for leg pain
D. Complication rates for the procedure are reasonabable
E. B and D are true
- Answer: D
Source: Feler C, Board Review 2005
764. In taking the pain history, what factors are critical to assess? A. Temporal features B. Expectational values of the patient C. Educational features of the patient D. All of the above E. None of the above
- Answer: A
Explanation:
The pain history should include temporal, provocative,
alleviative, and causative (initiative) parameters/ factors of
a particular patent’s pain.
Expectational values and educational features of the
patient may contribute to pain intensity, duration,
expression,and amenability to treatment, but are not
constituents of the pain history.
Source: Giordano J, Board Review 2005
- Of the following statements pertaining to post lumbar
puncture headaches, which is least accurate? Choose one:
A. Age, female gender, body mass index, and history of recurrent
headaches are major risk factors for PLPH.
B. 80% of the cases of PLPH occur within 48 hours of the
procedure.
C. 30 cc’s of CSF taken will not induce a headache by volume
loss and will be replaced within 90 minutes given
that CSF is produced at a rate of 1cc/3 minutes or approximately
500 cc’s a day.
D. 30 cc’s of CSF taken will likely induce a headache by
volume loss but will be replaced within 270 minutes
given that CSF is produced at a rate of 1cc/9 minutes or
approximately 150 cc’s a day.
E. Lying prone for 1 hour and drinking 24 ounces of water
after a LP has been shown to decrease the incidence of
PLPH.
- Answer: D
Source: Goodwin J, Board Review 2005
766. Personality disorders are easily diagnosed by: A. Careful history-taking B. Clinical interview C. Observing waiting room behavior D. Utilizing the Symptom Checklist – 90 E. None of the above
- Answer: C
Source: Janata J, Board Review 2006
767.The presence of pain behavior in chronic pain
presentations:
A. indicates that pain is “all in the patient’s head”
B. is a normal adaptation to an abnormal set of circumstances
C. is an indication of psychopathology
D. is abnormal in chronic pain populations
E. indicates the absence of true pathophysiology
- Answer: C
Source: Janata J, Board Review 2006
768. Münchhausen syndrome is a type of: A. Somatization disorder B. Factitious disorder C. Conversion disorder D. Hypochondriacal presentation E. Personality disorder
- Answer: B
Source: Janata J, Board Review 2006
769. The somatoform condition with the lowest incidence is: A. Malingering B. Somatization disorder C. Factitious disorder D. Conversion disorder E. Hypochondriasis
- Answer: D
Source: Janata J, Board Review 2006
770. Diagnostic of which plexus nerve should be performed for the testicular pain? A. Splanchnic nerves B. Lumbar sympathetic nerves C. Hypogastric plexus D. Genitofemoral nerve E. Ganglion impar
- Answer: A
Source: Day MR, Board Review 2005
771. Tietze’s syndrome is defi ned as unilateral costochondritis of what rib level/s? A. 1st and 2nd B. 2nd and 3rd C. 3rd and 4th D. 2nd only E. 3rd only
- Answer: B
Source: Day MR, Board Review 2006
- Which statement regarding occipital nerve stimulation
is true:
A. The electrode is placed transversely in the subcutaneous
tissue plane overlying C1-2.
B. The technique is contra-indicated in patients who have
undergone posterior cervical spine surgery.
C. Paresthesias are typically felt in the ipsilateral occiput
and down the ipsilateral arm
D. Bilateral occipital leads are contraindicated
E. A stimulation trial is not necessary prior to implant
- Answer: A
Explanation:
With occipital nerve stimulation, the lead is placed
transversely in the subcutaneous tissue plane overlying
C1-2, Unilateral or bilateral leads can be placed depending
on the patients’ pain pattern. Paresthesias are typically felt
in the occiput and sometimes in the posterior neck and
shoulder but do not radiate down the arm.The technique
is extraspinal so it can be used in patients’ who have
undergone previous posterior cervical spine surgery. As
with all neurostimulation, a successful trial of stimulation
is a necessary prerequisite for implant.
Reference:
Heavner, Interventional Pain Management, Second
Edition; Chapter 57 Peripheral Nerve Stimulation:
Current Concepts
Source: Schultz D, Board Review 2004
- Regarding meningitis, which of the following best
suggests meningeal irritation? Choose one:
A. Bilateral Horner’s syndrome
B. Inability to stay awake
C. A stiff neck coupled with Kernig’s and Brudzinski’s
signs
D. Inability to fall asleep because of headache-induced
nausea
E. Opisthotonus
- Answer: C
Source: Goodwin J, Board Review 2005
774. Self-effi cacy is synonymous with: A. Exclusive reliance on pain interventions B. External locus of control C. Internal locus of control D. Social support E. Euthymia
- Answer: C
Source: Janata JW, Board Review 2005
- Malingering involves production of false or exaggerated
symptoms that are:
A. Intentionally produced
B. Unconsciously motivated
C. Symptomatic of a psychotic process
D. Easily detectable on exam
E. Associated with family history of depression
- Answer: A
Source: Janata JW, Board Review 2005
776. Exclusion criteria for group therapy include all of the following EXCEPT: A. Severe depression B. Pain behavior C. Signifi cant personality disorders D. Capacity for violence E. Signifi cant history of noncompliance
- Answer: B
Source: Janata JW, Board Review 2005
- After heat radiofrequency lesioning of the right
sphenopalatine ganglion, the patient complains of right
upper tooth numbness. What is the likely explanation?
A. The greater palatine nerve was lesioned as well
B. The lesser palatine nerve was lesioned as well
C. The Vidian nerve was lesioned as well
D. The maxillary nerve was lesioned as well
E. The mandibular nerve was lesioned as well
- Answer: D
Source: Day MR, Board Review 2005
778. All of the following neurosurgical procedures for pain relief have historically been used for the treatment of psychiatric conditions except: A. Cingulotomy B. Anterior capsulotomy C. Leucotomy D. Hypothalamotomy E. Subtemporal sensory rhizotomy
- Answer: E
Explanation:
(Raj, Pain Review 2nd Ed., pages 309-311)
Cingulotomy, anterior capsulotomy (anterior limb of
internal capsule), leucotomy (pre-frontal lobotomy), and
hypothalamotomy have been used for intractable cancer
pain in multiple sites and for psychiatric disorders, such as
obsessive compulsive disorders. Hypothalamotomy, in
fact, may be benefi cial if there is a strong emotional
component to the pain. Subtemporal sensory rhizotomy
was the main operation performed for trigeminal
neuralgia before the 1950s’. Unfortunately, recurrence
rates ranged from 5-20% and there was a high incidence of
complications: anesthesia dolorosa, dyesthesias, keratitis.
Source: Schultz D, Board Review 2004
779. Which heating method is contraindicated in patients with spinal cord stimulation? A. Diathermy B. Hydrotherapy C. Heat Lamps D. Paraffi n E. Hot packs
779. Answer: A Explanation: Diathermy is contraindicated in patients with spinal cord stimulation Source: Shah RV, Board Review 2005
- Proper patient positioning for a subarachnoid phenol
block is:
A. Painful side up with no tilt
B. Painful side down with no tilt
C. Painful side up with the patient tilted anteriorly 45°.
D. Painful side down with the patient tilted posteriorly 45°.
E. Painful side down with the patient tilted anteriorly 45°.
- Answer: C
Source: Day MR, Board Review 2005
- In which of the following types of patients would
you expect the best results following a surgical
sympathectomy?
A. Failure of response to sympathetic blocks
B. Raynaud’s syndrome
C. Diabetic peripheral neuropathy
D. Phantom limb pain
E. Spinal cord injury end zone pain
- Answer: B
Explanation:
(Raj, Practical Management of Pain, page 803; Raj, Pain
Review 2nd Ed. Page 314).
All surgical sympathectomy should be prognosticated by a
series of sympathetic blocks which unequivocally give a
positive response. That being said, all of the above
disorders may have a sympathetically maintained
component. However, a painful vasospastic disorder such
asRaynaud’s or complex regional pain syndrome would do
the best. Central and chronic peripheral pain syndromes
are less predictable.
Source: Schultz D, Board Review 2004
- What is true about Tuffi er’s line?
A. It represents a horizontal line connecting the superiormost
aspects of the palpable iliac crests
B. It can be identifi ed, by using the inferior poles of the
scapulae as landmarks
C. It can be helpful in performing cervical epidural anesthesia
D. It is an imaginary line connecting the C7 and L5 spinous
processes
E. It represents needle trajectory during the performance of
a spinal anesthetic
- Answer: A
Explanation:
(Raj, Practical Mgmt of Pain, 3rd Ed., page 634)
Spinal anesthesia is usually instituted with a needle
inserted at an easily palpable interspace below L2.
Depending on the patient’s individual anatomical features,
the second, third, or fourth lumbar interspace may be
selected. After the most prominent point of the iliac crests
is located, an imaginary line is drawn between them
(Tuffi er’s line), which usually crosses the L4 spinous
process or the L4-L5 interspace.
Source: Shah RV, Board Review 2005
- The following are necessary of successful spinal cord
stimulation implant except:
A. Paresthesia sensation that overlaps region of pain
B. Comfortable paresthesia
C. Pain relief at low amplitudes
D. Intact cognitive abilities of the patient recipient
E. Absence of stimulation in nonpainful targets
- Answer: C
Explanation:
Reference:
Bedder, Interventional Pain Management, Second Edition;
Chapter 55 Implantation Techniques for Spinal Cord
Stimulation
Successful SCS implant requires a comfortable paresthesia
sensation that overlaps the area of pain. This should be
demonstrated in a trial of stimulation with the selected
device prior to a decision to implant. The patient should
have reasonably intact cognition because spinal cord
stimulation requires the patient to turn on and off the
device, keep track of the external programmer and be able
to manage a certain amount of technology. Many patients
with successfully implanted stimulators have paresthesia
sensation in areas outside the pain target region. This is
not necessarily a problem. Pain relief at low amplitudes is
desirable but not required since an RF system with an
external battery source can be used in these cases.
Source: Schultz D, Board Review 2004
784. A far left lateral disc bulge at the L4-5 level will likely affect which nerve root? A. The left L5 nerve root B. The left L4 nerve root C. The left L4 and L5 nerve roots D. The bilateral L4 nerve roots E. The bilateral L4 nerve roots
- Answer: B
Source: Bieneman B, Board Review 2005
- Which of the following regarding pontine spinothalamic
tractotomy is true?
A. A more caudal level of analgesia may be obtained compared
to a high cervical cordotomy
B. There is minimal risk of obstructive sleep apnea compared
to high cervical cordotomy
C. Pancoast tumor-related pain is not an indication
D. One theoretical advantage over mesencephalotomy is
that the neospinothalamic and paleospinothalamic
fi bers are in closer proximity
E. Oculomotor disturbances are common
- Answer: D
Explanation:
(Raj, Pain Review 2nd Ed., pages 310; Bonica 3rd Ed.,
pages 2048-9 & 2052-4;
Raj, Practical Mgmt of Pain 3rd Ed., pg 795)
Pontine spinothalamic tractotomy produces a more rostral
level of analgesia compared to a high cervical cordotomy.
Hence, it was introduced for neck and shoulder pain. C1-2
cordotomy at was among the most useful procedures for
unilateral cancer pain below the C5 dermatome. Both
pontine spinothalamic tractotomy and high cervical
cordotomy carry a similar risk of paralyzing the automatic
phase of respiration and thus, causing sleep apnea. Neck
and shoulder pain due to cancer, e.g., Pancoast tumor, are
indications for pontine spinothalamic tractotomy. The
neospinothalamic (spatial and temporal aspects of painful
stimuli and more laterally located) and paleospinothalamic
(affective and motivational aspects of painful stimuli and
more medially located)are closer together at the pontine as
compared to the midbrain (mesencephalotomy) level.
Oculomotor complications are common at the midbrain
level due to the proximity of the oculomotor nucleus and
medial longitudinal fasciculus. However,
mesencephalectomy does not carry a risk of sleep apnea.
Source: Schultz D, Board Review 2004
- For a patient presenting with a left facial droop and right
upper extremity paresis, the most likely site of the lesion
is:
A. Brainstem
B. Right parietal lobe
C. Origin of the left facial nerve and left motor cortex
D. Anterior bundle of the corpus collosum
E. Left frontal lobe
- Answer: A
Source: Goodwin J, Board Review 2005
787. Splenic laceration is best diagnosed by which of the following tests? A. Ultrasound B. Plain radiographs C. CT D. MR E. Sulfur colloid scan
- Answer: C
Explanation:
CT is the appropriate imaging modality in acute
abdominal emergencies and is preferred with IV contrast
for abdominal trauma
Source: Bieneman B, Board Review 2005
- Which of the following statements about nystagmus are true? Choose one:
A. Attenuating nystagmus may be a medication side effect
B. Immediate-onset nystagmus usually implicates inner ear
pathology
C. Delayed-onset nystagmus is a common cerebellar disease
fi nding
D. Nystagmus generally precedes and therefore heralds a neuromuscular junction disorder such as myesthenia
gravis, so one should initiate a search for a small cell
carcinoma of the lung.
E. A, B and C are correct
- Answer: A
Source: Goodwin J, Board Review 2005
789. Referred pain from pericarditis is felt where? A. Left shoulder B. Right shoulder C. Left upper quadrant of the abdomen D. Right upper quadrant of the abdomen E. Mid epigastrium
- Answer: A
Source: Day MR, Board Review 2006
- The 2004 International Headache Society’s revised
criteria for chronic tension- type headache (TTH),
requires a frequency over time consistent with which
one of the following:
A. 15 or more days per month over a minimum of 3
months
B. No more than 6 headache-free days in a 3 month period
C. No fewer than 2 days involvement per week for a minimum
of 3 months
D. Between 1 and 14 days per month over a 3 month period
E. An average of 60 days per year for at least 60 months (5
years
- Answer: A
Source: Goodwin J, Board Review 2005
- Current Perception Threshold testing:
A. Can evaluate small nerve fi bers impossible to assess on
standard EMG/NCS’s
B. Is of minimal value in the blind because visual perception
of stimuli is key to accuracy
C. Is very expensive to perform and therefore not widely
available
D. Is of little value in assessing pain and temperature
thresholds because the nerve fi bers are too small
E. Is of greatest value where axonal versus demyelinating
neuropathies need clarifi cation
- Answer: A
Source: Goodwin J, Board Review 2005
- Regarding plexopathies, which of the following is true?
A. Both the H-refl ex and F-waves may be prolonged
B. The H-refl ex and F-waves are usually normal
C. Fibrillations in paraspinal muscles do not rule in a
plexopathy, but are suggestive of it
D. Current perception threshold testing is less expensive
and more sensitive than EMG
E. Loss of an F-wave is meaningless because the action
potential is impeded by a proximal lesion and therefore
cannot be assessed with accuracy
- Answer: A
Source: Goodwin J, Board Review 2005
- The initial imaging modality recommended for evaluation
of traumatic odontoid (dens) fracture is?
A. X-ray tomograms of the odontoid process
B. Plain radiographs with lateral and open mouth views
C. CT scan with axial and coronal recontructions
D. T1-weighted MRI sagittal and coronal views
E. Triple phase bone scan to identify fracture line
- Answer: B
Source: Bieneman B, Board Review 2005
794. Nociception of the pancreas is mediated through which splanchnic nerves? A. T5-9 B. T10-11 C. T12 D. T8-11 E. T10-12
- Answer: A
Explanation:
Reference: Raj and Patt. Chapter 11. Visceral Pain. In:
Pain Medicine: A Comprehensive review, 2nd Edition. Raj,
Mosby, 2003, page 101.
Source: Day MR, Board Review 2005
- Which of the following does not characterize pelvic
congestion syndrome?
A. There is no identifi able pathologic condition
B. Pain is dull and achy
C. Complaints of suprapubic pain
D. May have psychosomatic features such as headaches and
urinary symptoms
E. Common in post-menopausal women
- Answer: E
Explanation:
Reference: Raj and Pott. Chapter 11. Visceral Pain. In Pain
Medicine: AComprehensive Review, 2nd Edition, Raj,
Mosby, 2003, page 102
Source: Day MR, Board Review 2005
796. All of the following are complications of a celiac/ splanchnic block except: A. Constipation B. Hypotension C. Paraplegia D. Pneumothorax E. Vascular injury
- Answer: A
Explanation:
Reference: Raj and Patt. Chapter 11. Visceral Pain. In:
Pain Medicine: AComprehensive Review, 2nd Edition, Raj,
Mosby, 2003, page 105
Source: Day MR, Board Review 2005
- True statements regarding Sacroiliac joint dysfunction
include all of the following except:
A. Pain radiating to hip, back, and thigh
B. Pain worsened by twisting movements
C. Straight leg raising may be positive
D. Pain worsened by sitting on the contralateral ischeal
tuberosity
E. May cause hamstring spasm
- Answer: D
Explanation:
Reference: Raj. Chapter 43. Thoracoabdominal Pain. In:
Practical Management of Pain.
3rd Edition. Raj et al, Mosby, 2000. page 627.
Source: Day MR, Board Review 2005
- When present, which of the following refl exes or signs
best localizes an upper motor lesion to a level above the
cervical spinal cord? Choose one:
A. Brisk jaw jerk
B. Babinski sign (upgoing toe)
C. Hoffman sign
D. Loss of the superfi cial abdominal refl exes
E. Clonus of one or both ankles
- Answer: A
Source: Goodwin J, Board Review 2005
799. The most appropriate imaging modality for acute headache is A. Magnetic resonance imaging B. Computed tomography C. Magnetic resonance angiography D. Intravenous angiography E. Duplex scanning
- Answer: B
Source: Bieneman B, Board Review 2005
800. What percentage of community-dwelling elderly suffer from chronic pain? A. 10-20% B. 20-25% C. 25-50% D. 50-60% E. > 60%
- Answer: C
Source: Day MR, Board Review 2006
- Classic features of a syrinx include:
A. Dissociated sensory loss
B. Long tract signs below the level of the lesion
C. Bowel or bladder dysfunction
D. All of the above
E. None of the above
- Answer: D
Source: Wirght PD, Board Review 2004
802. Shoulder shrug tests which nerve? A. Vagus B. CN X C. CN XII D. Accessory Nerve E. Phrenic Nerve
- Answer: D
Source: Wirght PD, Board Review 2004
803. The corneal refl ex tests the trigeminal nerve and: A. Vagus nerve B. Spinal accessory nerve C. Facial nerve D. Oculomotor nerve E. None of the above
- Answer: C
Source: Wirght PD, Board Review 2004
- The true statements about electromyography and nerve
conduction velocity:
A. Electromyographic changes occur within 24 h of neural
injury
B. Testing of neural conduction velocity is more sensitive
than electromyography in the early stages of neural
injury
C. Increased motor potential in muscle groups occurs with
neural injury
D. Increased neural conduction velocity occurs with neural
injury
E. Changes in neural conduction velocity take weeks to
become apparent after neural injury
- Answer: B
Explanation:
Reference: Tollison, p 326.
If neural injury is suspected, electromyography and testing
of neural conduction velocity can provide information as
to the extent and location of injury.
With neural injury, a decrease in motor potential in
muscle groups and slowed conduction velocities occur.
Neural conduction velocities are decreased quickly after
neural injury.
Electromyographic changes may take weeks to occur.
Therefore, testing of neural conduction velocity is more
sensitive than electromyography in the early stages of
neural injury.
Source: Kahn and Desio
- Spine myelography:
A. Contrast is instilled into the epidural space
B. Contrast is instilled into the subdural space
C. Contrast is instilled into the subarachnoid space
D. Myelography is not used for patients with MR contraindications
E. Myelography is preferable to MR for pregnant patients
- Answer: C
Source: Bieneman B, Board Review 2005
- In studying the interaction of psychological stress and
DNA repair, suppressed DNA repair was found in persons
with
A. Chronic stress more often than those with acute stress
B. Low distress over an extended period of time
C. A diagnosis of major depression
D. Increased anxiety and depression from bereavement
E. Recently diagnosed cancer
- Answer: C
Explanation:
(Baum, pp 194-198.)
·In studying peripheral blood lymphocytes (PBLs) from
patients with a major depression,it was found that they
had poorer DNA repair (PBLs exposed to radiation
damage) than lymphocytes obtained from nondepressed
or low-distressed persons. When patients with a diagnosis
of major depression were divided into low and highdistress
subgroups, the PBLs from the high-distress
subgroup had poorer DNA repair than the PBLs from the
low-distress subgroup.
·While acute stress is immunosuppressive, chronic stress
over time is associated with adaptation and can even
enhance immunity.
·It is hypothesized that the impact of psychosocial stress
(distress) on DNA repair could increase cancer risk. While
the impact of psychosocial stress and DNA repair on the
initiation of cancer has not been demonstrated, stressinduced
suppression of the immune system, and
enhancement of the immune system, has been shown to
affect the growth and progression of neoplasms.
·The increased anxiety and depression from bereavement
does produce suppressed lymphocyte proliferative
response to mitogen stimulation 2 to 6 weeks after the
death of a spouse.
Source: Ebert 2004
- A 67-year-old man with lung cancer presents with
metacarpophalangeal joint pain. On physical
examination, there is pain on moving his fi ngers and a
spongy sensation when palpating the proximal aspects of
the fi ngernails. CHOOSE ACCURATE DIAGNOSIS:
A. Refl ex sympathetic dystrophy
B. Ankylosing spondylitis
C. Reiter syndrome
D. Hypertrophic osteoarthropathy
E. Charcot joint
- Answer: D
Explanation:
(Goldman, 21/e, p 1558.)
Hypertrophic osteoarthropathy
is nail clubbing accompanied by a symmetrical
polyarthritis involving the large joints and occasionally
the metacarpophalangeal joints. Hypertrophic
osteoarthropathy may be seen secondary to malignancy,
endocarditis, vasculitis, and other pulmonary and cardiac
diseases. Ankylosing spondylitis (AS) is a chronic and
progressive infl ammatory disease, seen mostly in men in
their thirties, that most commonly affects the spinal,
sacroiliac, and hip joints. It may go undiagnosed for many
years, and bilateral hip pain due to sacroiliac involvement
may be clinically undetectable. It is strongly associated
with HLA-B27. Examination of the spine usually reveals
limitation in movement; patients in advanced stages may have a characteristic bent-over posture. Patients with AS
may present with an acute nongranulomatous uveitis and
limited chest expansion due to involvement of the
costovertebral joints. The Schober test is positive in AS
(with the patient erect, marks are made 5 cm below and 10
cm above the lumbosacral junction between the posterior
superior iliac spines; the patient bends, marks are
measured, and if the distance between the two marks
increases by less than 4 cm there is spinal immobility). The
pathogenesis of refl ex sympathetic dystrophy is unknown.
The presentation may be seen after peripheral limb injury;
early symptoms include pain in the limb and edema. This
disorder may lead to contractures. Charcot joint is a
complication of peripheral neuropathy seen in diabetic
patients. Repetitive minor trauma to the foot causes
deformities, which may lead to skin breakdown, erythema,
edema, and callus formation.
- Depletion of which neurotransmitter in the substania
nigra is associated with Parkinson’s disease?
A. Acetylcholine
B. Epinephrine
C. Calcitonin gene-related peptide
D. Dopamine
E. Substance P
- Answer: D
Source: Day MR, Board Review 2006
- A 42-year-old woman (5 ft, 3in., 170 lb) complains of
sudden onset of severe pain in the right upper abdomen
“under the ribs” accompanied by sweating, nausea, and a
feeling of imminent collapse. The pain lasts for about two
hours and then persists as a dull ache. When seen several
hours later, she has normal bowel sounds, is tender
throughout the abdomen, especially in the right upper
quadrant, and is faintly icteric. She has noticed her urine
is darker than usual but has not passed stool recently. She
recalls occasional episodes of “indigestion” referred to
the right upper abdomen and radiating to the shoulder.
This has occurred especially after eating fried foods or
after eating a meal following a long period of fasting.
She has no fever but is anxious and tachycardic.The
tests available are a blood count and blood chemistry
including liver enzymes, alkaline phosphatase, and
bilirubin. She has a WBC of 10,000. Her cellular hepatic
enzymes are: AST/SGOT = 52 (2-33) and ALT/SGPT = 70
(4 to 44), alkaline phosphatase = 300 (17 to 91), bilirubin
= 6.3 (0.2 to 1.0).The most probable diagnosis is
A. Hepatitis A
B. Intercostal neuritis
C. Carcinoma of the head of the pancreas
D. Gallstone obstructing common bile duct
E. Biliary cirrhosis
- Answer: D
Explanation:
(Braunwald, 15/e, pp 255-259, 1785. Kumar, 6/e, pp 550- - Junqueira, 9/e, pp 318-319. Guyton, l0/e, pp 800-801.)
The most probable diagnosis is gallstones. The pattern
of elevated liver enzymes, alkaline phosphatase, and
bilirubin are consistent with obstructive jaundice (see
table below). The presence of pain (in the right upper
quadrant radiating to the shoulder) after eating a meal
consisting of fried foods makes gallstones the most
probable diagnosis. Similar pain often occurs in these
patients when they have not eaten for long periods of time
and then have a large meal. The pain is caused by the
obstruction of the cystic duct or common bile duct that
produces increased lumenal pressure within the bile
vessels, which cannot be compensated for by
cholecytokinin-induced contractions. The pain lasts for
about one to four hours as a steady, aching feeling.
Source: Klein RM and McKenzie JC 2002.
- Which of the following words is defi ned as A chronic
preoccupation with obtaining the substance of choice
and misuse or overuse of the substance despite negative
consequences?
A. Tolerance
B. Physical dependence
C. Pseudo addiction
D. Psychological dependence
E. Addict
- Answer: D
Source: Day MR, Board Review 2006
811. What is the most frequent initial site of metastatic tumor spread to the spine? A. Bone marrow B. Vertebral pedicle C. Nucleus pulposus D. Epidural space E. Posterior elements
- Answer: A
Explanation:
Because of its high vascularity, bone marrow is involved
fi rst, with extension to pedicle and posterior elements.
Extension to epidural space may occur from vertebra or
through foramen.
Source: Bieneman B, Board Review 2005
812. Which of the following type A, or coronary-prone, behavioral factors appears to be the best predictor of coronary heart disease? A. Hostility B. Competitiveness C. Time Urgency D. Explosive speech E. Hyperactivity
- Answer: A
Explanation:
(Baum, pp 144-148.)
Among the psychosocial variables
considered to be risk factors for coronary heart disease, the type behavior pattern is most prominent. The type A
behavior pattern is most prominent. The type A behavior
pattern consists of extremes of competitiveness, a chronic
sense of time urgency, easily evoked hostility,
aggressiveness, explosive speech, and increased rate of
activity. More recent studies have shown that
aggressiveness and hostility (especially unexpressed
hostility) are the most consistent and important factors
Source: Ebert 2004
813. The most common cause of peripheral neuropathy is: A. Idiopathic B. Diabetes Mellitus C. Nutritional Defi ciencies D. ETOH E. None of the above
- Answer: B
Source: Wirght PD, Board Review 2004
814. In the clinical assessment of neuropathic pain, which procedure(s) should be included in the diagnostic workup? A. EMG and/or NCV B. Laboratory evaluations C. Imaging studies D. All of the above E. None of the above
- Answer: D
Source: Giordano J, Board Review 2003
- The best defi nition for a vertebral disc bulge is which of
the following?
A. An intraverterbral disk herniation (Schmorl’s node)
B. Disruption of concentric fi bers of the annulus fi brosis
C. Generalized extension of disc material beyond the edge
of the vertebra involving 180°
D. Localized displacement (
- Answer: C
Source: Bieneman B, Board Review 2005
- A complete electrodiagnostic evaluation would include
all the following EXCEPT
A. Electromyography and late response studies
B. Peripheral nerve conduction studies of motor and sensory
nerves
C. Muscle biopsy
D. Somatosensory evoked potentials
E. Radiologic evaluation
- Answer: C
Explanation:
Reference: Bonica, p 629.
The electromyogram (EMG), peripheral nerve conduction
studies (NCSs), late response studies, and somatosensory
evoked potentials (SEPs) help to characterize the nature
and location of the abnormality being studied.
Determination of the cause of the abnormality can occur
only after integration of the information obtained from
the physical examination, history, and electrodiagnostic
and radiologic studies.
Muscle biopsy is not a component of electrodiagnostic
evaluation.
Source: Kahn and Desio
817. A celiac-plexus block would not effectively treat pain resulting from a malignancy involving the following organs: A. Ureter B. Adrenal gland C. Stomach D. Pancreas E. Gallbladder
- Answer: A
Explanation:
The celiac-plexus innervates most of the abdominal
viscera, including the pancreas, liver, spleen, kidneys,
adrenal glands, biliary tract, omentum, and small and large
bowel.
The pelvic organs are supplied by the hypogastric
plexus.
- Traditional psychotherapy emphasizes all of the following
EXCEPT:
A. Formation of defense mechanisms
B. Psychosexual development
C. The role of environmental reinforcement
D. The therapeutic process of transference
E. The relationship between conscious and unconscious
processes
- Answer: C
Source: Janata JW, Board Review 2005
- Positive reinforcement refers to:
A. A stimulus that increases the likelihood that a certain
behavior will be maintained or repeated
B. A consequence that decreases the likelihood that a certain
behavior will be maintained or repeated
C. A consequence that increases the likelihood that a certain behavior will be maintained or repeated
D. A stimulus that decreases the likelihood that a certain
behavior will be maintained or repeated
E. A consequence that does not infl uence the likelihood that
a certain behavior will be maintained or repeated
- Answer: C
Source: Janata JW, Board Review 2005
820. Which of the following medications does not potentiate opioid respiratory depression? A. Dexmetetomidine B. Methohexital C. Etomidate D. Diazepan E. Propofol
- Answer: A
Source: Day MR, Board Review 2005
- Understanding the complex interaction of a patient’s pain
and mood is best accomplished by:
A. Integrating medical and psychological data from a variety
of sources
B. Performing a pain tolerance test in the laboratory
C. Having the patient complete the Michigan Pain States
Inventory (MPSI)
D. Asking offi ce staff to observe waiting room pain behavior
E. Obtaining family history of alcoholism
- Answer: A
Source: Janata JW, Board Review 2005
822. Personality disorders can be readily assessed by: A. Clinical interview B. Careful history-taking C. Utilizing the Symptom Checklist - 90 D. Observing waiting room behavior E. MMPI or MCMI
- Answer: E
Source: Janata JW, Board Review 2005
- A “fake bad” profi le can be estimated by using the:
A. Medical Outcomes Survey (MOS)
B. Sickness Impact Profi le (SIP)
C. Minnesota Multiphasic Personality Inventory (MMPI)
D. Coping Strategies Questionnaire
E. Spielberger State-Trait Anxiety Inventory (STAI)
- Answer: C
Source: Janata JW, Board Review 2005
- Which of the following statements regarding the superior
hypogastric plexus block is not true?
A. It is most appropriate for pelvic pain of visceral origin
B. It is associated with few side effects
C. It must be performed with the assistance of fl uoroscopy
D. It is most appropriate for upper abdominal pain
E. It must be performed at L5
- Answer: D
Source: Raj P, Pain medicine - A comprehensive Review -
Second Edition
- A 30-year old patient presents with foot pain. She was
diagnosed with a calcaneal heel spur. Non-steroidal antiinfl
ammatory agents failed to provide her any signifi cant
relief. Appropriate treatment is:
A. Soft padding of the shoe
B. Local steroid injection
C. Strengthening exercises
D. Surgical excision of spur
E. Stretching exercises in combination with ultrasound
- Answer: A
Explanation:
Goals of therapy include controlling the abnormal
biomechanics of the foot, decreasing the infl ammatory
condition, and improving the fl exibility. Various
modalities of treatments mentioned include the
following:
Non-steroidal anti-infl ammatory agent
Rest
Night splint
Padding
Physical therapy with stretching and strengthening
exercises
Physical therapy with ultrasound
Orthosis
Steroid injection
Surgical removal
- Spinal Shock:
A. Occurs weeks to months after initial injury
B. Is frequently associated with autonomic dysfunction
C. Will usually result in full recovery of function
D. All of the above
E. None of the above
- Answer: B
Source: Wirght PD, Board Review 2004
- The Mutidimensional Pain Inventory (MPI):
A. Is a projective test
B. Assesses malingering
C. Yields depression and anxiety scores
D. Utilizes three profi le types - Dysfunctional, Interpersonally
Distressed and Adaptive Coper.
E. Contains 576 true-false questions
- Answer: D
Source: Janata JW, Board Review 2005
828. Factitious disorder is motivated by: A. Secondary gain B. Assumption of the sick role C. Financial reward D. Evasion of responsibility E. External incentive
- Answer: B
Source: Janata JW, Board Review 2005
829. Which of the following is most appropriate for the initial treatment of tension headache? A. Acetaminophen B. Amitriptyline C. Gabapentin D. Oxycodone E. Tramadol
- Answer: A
- A thoracic epidural is placed at T10 after abdominal
surgery. A test dose is given and the patient becomes
numb above the incision for 2 hours. An epidural
catheter is inserted and an infusion is started. After 24
hours, she develops abdominal pain over 30 minutes.
Your next course of action is:
A. Notify surgeon
B. Give IV NSAIDs
C. Give IV morphine
D. Test the epidural
E. Apply TENS unit
- Answer: D
- A 41-year-old man was recently in a motor vehicle
accident (MVA) where he was the driver. He states he was
wearing his seat belt at the time of the accident. A day
after the accident, he developed neck pain that has now
continued for 10 days. He notices crunching on extension
and lateral bending of the neck. On physical examination,
the patient has no neurologic defi cits. His neck has no
areas of tenderness and there are no areas of spasm. He
has normal lateral bend, extension, and fl exion of the
neck. Which of the following is the most likely diagnosis?
A. Ankylosing spondylitis
B. Osteoarthritis
C. Reiter syndrome
D. Whiplash
E. Wry neck
- Answer: D
Explanation:
(Tierney, 42/e, p 792.)
The most likely diagnosis in this patient is whiplash or
cervical musculoligamental sprain or strain. Whiplashassociated
disorders begin after a symptom-free period
following a hyperextension or hyperfl exion injury, usually in an MVA. It is vital to perform a complete neurologic
examination to exclude other causes of neck pain.
Ankylosing spondylitis is a chronic and progressive
infl ammatory disease that most commonly affects spinal,
sacroiliac, and hip joints.
Osteoarthritis most often affects the weight-bearing joints.
Reiter syndrome usually causes an arthritis of the hips, and
there is often a history of urethritis, conjunctivitis, and
foot involvement.
832. The appropriate initial treatment for mild mandibular pain following oral surgery is A. Nonsteroidal antiinfl ammatory drug B. Mandibular nerve block C. Acetaminophen D. Oxycodone E. Gabapentin
- Answer: A
- A 29-year old female with upper extremity complex
regional pain syndrome undergoes a stellate ganglion
block in your offi ce pain clinic. She is otherwise healthy
with normal body habitus and normal airway. She has
been NPO for 12 hours. 20cc of 0.25% bupivacaine is
injected incrementally over one minute with no other
medication administered. 5 minutes after injection
the patient complains of generalized weakness which
progresses to complete unresponsiveness, apnea and
hypotension over the ensuing several minutes. Eight
minutes after injection, the patient continues to be
completely unresponsive and apneic with a systolic blood
pressure of 50. ECG monitor shows sinus bradycardia.
The patient has no IV. Your fi rst action should be:
A. Start an IV
B. Use an ambu bag to ventilate the patient
C. Intubate the patient
D. Administer subcutaneous epinephrine
E. Begin CPR
- Answer: B
Explanation:
In resuscitation scenarios, always remember the ABCs:
airway
breathing
circulation
Immediate control of the airway is the most important
and pressing concern in any arrest situation. This patient is
completely apneic therefore airway management takes
precedence over circulatory compromise. Since the patient
has a normal airway and has been NPO, there is no
immediate need to intubate if the airway can be
maintained easily with a mask and ambu bag. The patient
will likely regain the ability to ventilate on her own in a
relatively short period of time (1-2 hours).Intubation
in this patient would introduce a number of additional
concerns including the potential for airway damage, the
potential for bronchospasm, and issues related to timing
of extubation.
Intravenous access and epinephrine are circulation
interventions. Starting an intravenous line is important
but only after the airway is secure. Epinephrine is an
extremely potent sympathomimetic and should be used
very cautiously. It has the potential to cause severe
hypertension and tachycardia in a patient who is not in
full cardiac arrest (when the patient is in full cardiac arrest,
epinephrine is indicated per ACLS guidelines). This
patient is hypotensive and bradycardic secondary to
sympathetic blockade. Intravenous volume perhaps with
the edition of a milder vasopressor such as ephedrine is a
better choice than epinephrine.
Source: Schultz D, Board Review 2004
834. Tissues that are more echogenic on sonography: A. Appear darker B. Include cysts C. Have more refl ective surfaces D. Do not include fat E. Include moving blood
- Answer: C
Source: Bieneman B, Board Review 2005
835. Mixed Upper and Lower Motor Neuron fi ndings can be caused by: A. Vit B12 defi ciency B. Nitrous Oxide Exposure C. Cervical Spinal Stenosis D. All of the above E. None of the above
- Answer: D
Source: Wirght PD, Board Review 2004
836. A L4-5 left paracentral disc protrusion will likely affect which nerve root? A. The left L5 nerve root B. The left L4 nerve root C. The left L4 and L5 nerve roots D. The bilateral L5 nerve roots E. The bilateral L4 nerve roots
- Answer: A
Source: Bieneman B, Board Review 2005
- Which of the following is the most appropriate initial
examination to evaluate for disc herniation?
A. Enhanced MR
B. Unenhanced MR
C. Enhanced CT
D. Unenhanced CT
E. CT Myelogram
- Answer: B
Source: Bieneman B, Board Review 2005
838. Aortic dissection is best diagnosed by what test? A. Unenhanced CT B. Ultrasound C. Unenhanced and enhanced CT D. MRI E. Angiography
- Answer: C
Explanation:
Aortic dissection is an acute abdominal emergency and
best imaged with CT due to the speed of examination and
ability to characterize the type of dissection without delay.
MRI and ultrasound may provide appropriate imaging of
an aortic dissection in the non acute setting. Angiography
is reserved for cases with clinical questions not answered
by the initial imaging modality and for cases where further
intervention is planned such as fenestration of a dissection
of placement of a stent graft for aortic aneurysm
Source: Bieneman B, Board Review 2005
839. Increased activity on bone scintigraphy may be from all except: A. Healing fracture B. Prostate metastases C. Stress fracture D. Interrupted sympathetic nerve supply E. Old orthopedic hardware
- Answer: E
Source: Bieneman B, Board Review 2005
840. Vertebral discitis and osteomyelitis are best imaged by A. MRI B. Bone scan C. CT-myelogram D. Plain radiographs E. Duplex ultrasound
- Answer: A
Source: Bieneman B, Board Review 2005
- Bilateral cingulumotomy is a properly selected therapy in nociceptive back pain.
A. If the patient has no obsessive compulsive feature
B. In cancer patients who have no other option
C. In most benign pain patient
D. If the patient hass failed oral opiate analgesics and tricyclic
antidepressant
E. Two of the above
- Answer: B
Source: Feler C, Board Review 2005
842. Tissues that appear denser on plain radiographs have: A. More electron density B. Less electron density C. More neutron density D. Less neutron density E. None of the above
- Answer: A
Source: Bieneman B, Board Review 2005
- Which of the following statements about DREZ lesions
is correct?
A. DREZ is currently recommended in the treatment of
PHN
B. DREZ lesions typically produce sensory loss that improves
with time
C. The techniques used to create DREZ lesions include
ultrasonic aspiration
D. Results of DREZ lesioning are predictable in all neuropathic
pain state
E. All of the above
- Answer: B
Source: Feler C, Board Review 2005
844. Houndsfield units on CT: A. Are a measure of enhancement B. Water=200 HU C. Acute hemorrhage = -100 HU D. Are a measure of density E. Air=5 HU
- Answer: D
Explanation:
(See lecture notes)
Source: Bieneman B, Board Review 2005
- In the fi eld of electromyography:
A. The blink refl ex assesses the integrity of peripheral and
central circuitry involving CN5 and CN7
B. Presence of a blink refl ex is suggestive of frontal lobe
dementia
C. Repetitive stimulation is a technique unlikely to detect
pathology at the NMJ
D. Myopathies tend to be characterized by a reduced pattern
of recruitment
E. Neuropathies tend to be characterized by an early pattern of recruitment
- Answer: A
Source: Goodwin J, Board Review 2005
- The following statements regarding TENS are true
except:
A. It has been used very successfully in the treatment of
acute pain.
B. It works by stimulating A-beta fi bers which in turn close
the dorsal horn “gate” to nociceptive input.
C. TENS has been shown to increase blood fl ow in the
stimulated region
D. The effects of TENS occur only during the stimulation
period
E. Conventional TENS uses a nearly continuous high frequency
stimulation (60-100 Hz) and a relatively low
intensity current (10-30 milliamps)
- Answer: D
Explanation:
Reference:
Bonica’s Management of Pain, Third Edition, Chapter 98,
Transcutaneous Electrical Nerve Stimulation
TENS works by stimulating A-beta fi bers which in turn
close the dorsal horn “gate” to nociceptive input. It has
been shown in many well-controlled studies to be highly
effective for acute pain. The results of TENS for chronic
pain have been less impressive although there is data to
suggest that TENS is helpful for well selected patients with
chronic pain. Conventional TENS uses a nearly
continuous high frequency stimulation (60-100 Hz) and a
relatively low intensity current (10-30 milliamps). The
effects of TENS have been shown to increase blood fl ow in
the stimulated area with pain relief sometimes outlasting
the period of stimulation by hours to days.
Source: Schultz D, Board Review 2004
847. Which of the following is most appropriate for a patient with end-stage rectal cancer? A. Lissauer tractotomy B. Cingulotomy C. Hypophysectomy D. Commisural myelotomy E. Percutaneous C1-2 cordotomy
- Answer: D
Explanation:
(Raj 2nd Ed., page 313)
Midline or commissural myelotomy sections those
midline fi bers just dorsal to the central canal of the spinal
cord. The original intent was to lesion crossing
spinothalamic neurons, which would eliminate pain, but
preserve sensory function. However, pain relief extended
caudally, without demonstrable caudal analgesia. This lead
several investigators to postulate several alternate pain
pathways. A multisynaptic short tract afferent pathway or
an anterior tract located in between the posterior columns
were proposed. The latter mediate pelvic and epigastric
visceral pain. Nonetheless, myelotomy is indicated for
bilateral pelvic and perineal pain of malignant origin.
Unilateral percutaneous cordotomy is among the most
useful procedures for unilateral cancer pain below C5. It
targets the spinothalamic tract. Radiofrequency energy is
used. Electrical stimulation (sensory to obtain a feeling of
warmth or coolness on the contralateral side and motor to
obtain ipsilateral cervical muscles; ipsilateral contraction
of muscles below the neck implies the probe is in the
corticospinal tract) is used to identify the lesion target
Lissauer tractotomy is the goal of the dorsal root entry
zone (DREZ) procedure…but all dorsal horn lamina (I-V)
may be affected. The DREZ lesion is classically indicated
for central nervous system damage related pain: brachial
plexus avulsion, stump pain, spinal cord injury pain.
Hypophysectomy is recommended in the treatment of
metastatic prostate and breast cancer, irrespective of the
hormonal responsiveness of the tumors. The analgesic
mechanism is unknown, but limbic system or
psychological effects are unlikely to be the reasons for pain
relief.
Cingulotomy, anterior capsulotomy (anterior limb of
internal capsule), leucotomy (pre-frontal lobotomy), and
hypothalamotomy have been used for intractable cancer
pain in multiple sites and for psychiatric disorders, such as
obsessive compulsive disorders.
Source: Schultz D, Board Review 2004
- Deep brain stimulation to treat primarily nociceptive
pain would most likely target the:
A. Periaqueductal grey
B. Ventroposterolateral or Ventroposteromedial thalamus
C. Caudalis subnucleus
D. Nucleus gracilis
E. Reticular formation
- Answer: A
Explanation:
(Raj, Pain Review 2nd Ed., pages 311; Bonica 3rd Ed.,
pages 122, 130-2, & 153-4, ;
Raj, Practical Mgmt of Pain 3rd Ed., pg 795)
The periaqueductal and periventricular grey are located in
the midbrain. The PAG and PVG can be excited by
endogenous opioids or electrical stimulation to initiate
descending antinociception. The VPM and VPL located in
the thalamus are useful for deafferentation or neuropathic
pain. The caudalis subnucleus, also known as the
trigeminal spinal nucleus, is thought to be integral for a
variety of head pain syndromes. The reticular formation
may be responsible for some of the affective and
motivational responses to pain and the regulation of spinal
motor, respiratory, and autonomic functions: arousal and
escape. Pos-synaptic touch and proprioception fi bers (dorsal column) project to the dorsal column nuclei:
cuneatus and gracilis: lumbar and thoracic fi bers to the
gracilis and cervical to the cuneatus.
Source: Schultz D, Board Review 2004
- All of the following are typically associated with the
technical aspects of epidural anesthesia, except:
A. Paramedian
B. Bromage grip
C. Hanging drop
D. Taylor approach
E. Sacral hiatus.
- Answer: D
Explanation:
(Raj, Practical Mgmt of Pain, 3rd Ed, pages 641-6, 634)
A. Midline and paramedian approaches to the epidural
space have been described.
B. The Bromage grip is a useful technique for slow,
controlled advancement of an epidural needle towards the
ligamentum fl avum.
The needle is fi rmly gripped between the thumb and
index fi nger of the nondominant hand.
The dorsum of the wrist is placed against the patient’s
back.
The needle is advanced by extension of the wrist while
the dominant hand provides intermittent or constant
pressure on the plunger, depending on whether one uses
the loss-of-resistance technique to air or to saline solution,
respectively.
C. The negative pressure often found within the epidural
space is the basis for the hanging-drop technique.
This hanging-drop sign of Gutierrez is used to
identify the epidural space and is usually applied
for cervical epidural blockade in the seated patients.
A winged needle is usually used and is advanced with
both hands, as with the intermittent technique.
A drop of fl uid is placed at the end of the needle once
it is anchored in the interspinous ligament.
Because of the persistent subatmospheric pressure
within the epidural space, penetration of the ligamentum
fl avum and entrance of the epidural space cause the drop
to be sucked into the hub of the epidural needle.
Injection of air or fl uid without resistance confi rms
the position in the epidural space.
D. The Taylor is used to identify the subarachnoid space by
way of the L5 interspace, which is the largest interspace in
the vertebral column.
To enter this space, the operator introduces the spinal
needle through the skin wheal approximately 1cm medial
and 1cm inferior to the posterior superior iliac spine.
The spinal needle is directed medial and cephalad to
enter the subarachnoid space at the midline at the L5-S1
interspace.
E. Identifi cation of the sacral hiatus is important for
caudal epidural procedures.
The sacral hiatus is formed secondary to nonfusion of
the fi fth sacral vertebral arch.
The hiatus is covered by the sacrococcygeal membrane and
bordered by two cornua (large bony processes on each side
of the hiatus).
The sacral hiatus is most easily identifi ed with the
patient lying in the prone or lateral position.
Firm pressure is used to identify the coccyx with the
nondominant index fi nger.The fi rst pair of bony
protuberances in moving cephalad are the two cornua,
surrounding the sacral hiatus.
Source: Shah RV, Board Review 2005
850. Back pain developing after spine surgery is best imaged by A. Unenhanced MR B. Contrast Enhanced MR C. Unenhanced CT D. Enhanced CT E. Myelography
- Answer: B
Source: Bieneman B, Board Review 2005
- What is the co-morbid condition of body dysmorphic
disorder?
A. Depression
B. Multiple recurrent somatic complaints without medical
findings
C. Delusion
D. “La belle indifference”
E. Fear of having a serious illness despite adequate medical evaluation
- Answer: C
Explanation:
(A) Major depression is a comorbid condition of both
somatization disorder and hypochondriasis, but not a
major diagnostic feature.
(B) Somatization disorder is characterized by the
recurrence of multiple somatic complaints not accounted
for by medical fi ndings. It is a chronic condition with
female predominance.
(C) Delusion is the common feature of body dysmorphic
disorder.
(D) La belle indifference is an associated feature of
conversion disorder, where symptoms do not conform to
anatomic pathways. Delusional disorder may be a
comorbid condition in body dysmorphic disorder.
(E) Hypochondriasis is a chronic condition characterized
by a fear or belief that one has a serious illness despite
adequate medical evaluation. Its prevalence is 4% to 9% of
medical outpatients with equal incidence between men
and women.
Source: Laxmaiah Manchikanti, MD
852. The dorsal columns of the spinal cord primarily carry: A. Pain sensation B. Temperature sensation C. Spinothalamic tracts D. All of the above E. None of the above
- Answer: E
Source: Wirght PD, Board Review 2004
853. In a patient with midthoracic back pain who reports tenderness to palpation over the T-6 vertebral body, the most likely diagnosis is: A. Thoracic disk herniation B. Metastatic neoplasm C. Facet osteoarthropathy D. Rheumatoid arthritis E. Epidural hematoma
- Answer: B
Explanation:
Local spine tenderness elicited when palpating directly
over the vertebral body is highly suggestive of vertebral
body neoplasm or infection. Neoplastic conditions or
infectious-infl ammatory disorders (osteomyelitis) may
distend the periosteum, causing local tenderness. This
discrete local tenderness should be differentiated from
more diffuse muscle spasm seen with a herniated disk.
Source: Neurology for the Psychiatry specialty Board
Review By Leon A. Weisberg, MD
854. Which of the following sedative - hypnotic medications should not be used in a patient with increased intracranial pressure? A. Thiopental B. Etomidate C. Ketamine D. Propofol E. Midazolam
- Answer: C
Source: Day MR, Board Review 2006
- A “fake bad” profile is provided in scoring the:
A. Spielberger State-Trait Anxiety Inventory (STAI)
B. Minnesota Multiphasic Personality Inventory (MMPI)
C. Coping Strategies Questionnaire
D. Medical Outcomes Survey (MOS)
E. Sickness Impact Profi le (SIP)
- Answer: C
Source: Janata J, Board Review 2006
- Which of the following is not defi ned as a disc
herniation?
A. Protrusion
B. Localized displacement of disc material beyond the confi
nes of the disc space
C. Bulge
D. Extrusion
E. Intravertebral end plate disruption secondary to disc
material
- Answer: C
Explanation:
A disc bulge is not a herniation
Source: Bieneman B, Board Review 2005
- Understanding the complex interaction of a patient’s
pain and mood is best accomplished by:
A. Obtaining family history of depression
B. Asking offi ce staff to observe waiting room pain behavior
C. Having the patient complete the Michigan Pain States
Inventory (MPSI)
D. Integrating medical and psychological data from a variety
of sources
E. Performing a pain tolerance test in the laboratory
- Answer: E
Source: Janata J, Board Review 2006
- Mesencephalic tractotomy is indicated in patients who
suffer from the pain of head and neck cancer.
A. If there is no preoperative neurologic defi cit
B. If the neck pain does not come higher than the C5 segment
C. If the patient has a preoperative myelopathy
D. If the pain involves the lower face
E. Two of the above
- Answer: E
Source: Feler C, Board Review 2005
859. Which of the following drugs shows a good correlation between the blood level and the clinical effect? A. Phenelzine B. Trazodone C. Fluoxetine D. Paroxetine E. Imipramine
- Answer: E
Explanation:
Blood level can be obtained for all antidepressant drugs.
But not all of them have shown a correlation between the
therapeutic effect and the blood level.
Plasma level measurements of imipramine,
desmethylimipramine, and nortriptyline are unequivocally
clinically useful in certain situations. For imipramine, the
percentage of favorable responses correlates with plasma
levels in a linear manner between 200 and 250 ng/mL, but
some patients may respond at a lower level. At levels that
exceed 250 ng/mL, there is no improved favorable
response, and side effects increase.
Source: Laxmaiah Manchikanti, MD
- Some physiological consequences of epidural blockade
may include:
A. increased peristalsis
B. interference with satiety
C. impaired respiratory function, by modifying respiratory
drive, reducing diaphragmatic contractility, permitting
increased airway hyperactivity, and impairing V/Q
mismatch
D. hypertension
E. degree of sympathetic blockade that correlates with the
degree of sensory blockade
- Answer: A
Explanation:
(Raj, Practical Mgmt of Pain, 3rd Ed, pages 639-41)
A. The gastrointestinal tract is innervated by both the
sympathetic and parasympathetic systems.Visceral afferent
parasympathetic fi bers transmit sensations of satiety,
distention, and nausea (but not pain).
Parasympathetic efferent outfl ow increases tonic
contraction, sphincter tone, peristalsis, and secretions.
Pain is mediated via sympathetic afferents, whereas
sympathetic efferent fi bers inhibit peristalsis and gastric
secretions, constrict vasculature, and increase sphincter
tone.
Sympathetic denervation of the gastrointestinal tract
by neuroaxial blockade conceivably may lead to
generalized contraction of the bowel secondary to
unopposed parasympathetic efferent outfl ow.
The degree to which the bowel is affected after neural
blockade depends on the extent of the blockade.
C. It is conceivable that a high thoracic or cervical block
may impair respiratory function by affecting sensory
function (modifying respiratory drive), motor function (decreasing abdominal muscles, intercostal muscles, and
diaphragmatic strength), and sympathetic function
(unopposed cholinergic tone can lead to hyperreactive
airways).
Sympathetic block may diminish pulmonary blood fl ow
and ventilation-perfusion ( V/Q ) mismatch.
All of these changes have the potential to lead to airway
closure, atelectasis, decreased blood fl ow, and diminished
functional reserve capacity (FRC), causing (V/Q)
mismatch and hypoxemia.
Experimental evidence, however, does not support this
scheme.
D. The principal cardiovascular consequences of extensive
epidural blockade are hypotension and bradycardia.
E.The extent of sympathetic blockade, however, correlates
very poorly with the sensory level. The amount of
sympathetic denervation and sensory blockade might be
larger than the classically taught two levels.
It has been demonstrated that the sympathetic block
could extend six or more spinal segments above the level
of sensory blockade.
Source: Shah RV, Board Review 2005
- What is the major diagnostic feature of somatization
disorder?
A. Depression
B. Multiple recurrent somatic complaints without medical
fi ndings
C. Delusion
D. “La belle indifference”
E. Fear of having a serious illness despite adequate medical evaluation
- Answer: B
Explanation:
(A) Major depression is a comorbid condition of both
somatization disorder and hypochondriasis, but not a
major diagnostic feature.
(B) Somatization disorder is characterized by the
recurrence of multiple somatic complaints not accounted
for by medical fi ndings. It is a chronic condition with
female predominance.
(C) Delusion is not a common feature of either
somatization disorder or hypochondriasis.
(D) La belle indifference is an associated feature of
conversion disorder, where symptoms do not conform to
anatomic pathways. Delusional disorder may be a
comorbid condition in body dysmorphic disorder.
(E) Hypochondriasis is a chronic condition characterized
by a fear or belief that one has a serious illness despite
adequate medical evaluation. Its prevalence is 4% to 9% of
medical outpatients with equal incidence between men
and women.
Source: Laxmaiah Manchikanti, MD
- Which one of the following drugs causes Grand mal
seizure as the most prominent side effect?
A. Venlafaxine (Effexor®)
B. Phenelzine (Nardil®)
C. Fluoxetine (Prozac®)
D. Amitriptyline (Elavil®)
E. Bupropion (Wellbutrin®)
- Answer: E
Explanation:
Bupropion is associated with grand ,al seizures in
approximately 0.4% (4/1000) of patients treated at doses
up to 450 mg/day.This incidence of seizures may exceed
that of other marketed antidepressants by as much as 4-fold. This relative risk is only an approximate estimate
because of the lack of direct comparative studies. The
estimated seizures incidence for Bupropion increases
almost 10-fold between 450 and 600 mg/day, which is
twice the usually required daily dose (300 mg).
Source: Laxmaiah Manchikanti, MD
863. Electroconvulsive therapy is least likely to be successful in which of the following diseases? A. Major depression B. Acute schizophrenia C. Acute manic episodes D. Chronic schizophrenia E. Obsessive-compulsive disorder
- Answer: D
Explanation:
Catatonia, mania, major depression, and acute
schizophrenia are established indications of
electroconvulsive therapy (ECT). Other indications of
electroconvulsive therapy with less evidence of its
effectiveness include Parkinson disease, obsessivecompulsive
disorder, neuroleptic malignant syndrome,
and intractable epilepsy
Source: Laxmaiah Manchikanti, MD
864. The CAGE questionnaire is used in case of A. Mental retardation B. Bipolar disorder C. Major depression D. Opioid abuse E. Alcohol abuse
- Answer: D
Explanation:
Four clinical interview questions, the CAGE questions,
have proved useful in helping to make a diagnosis of
alcoholism. The questions focus on Cutting Down,
Annoyance by Criticism, Guilty Feeling, and Eye-Openers.
The acronym “CAGE” helps the physician recall the
questions:
“C”: Have you ever felt you should cut down on your
drinking?
“A”: Have people annoyed you by criticizing your
drinking?
“G”: Have you ever felt bad or guilty about your drinking?
“E:: Have you ever had a drink fi rst thing in the morning
to steady your nerves or to get rid of a hangover?
Source: Laxmaiah Manchikanti, MD
- Cordotomy is most useful in treating patients with pain
complaints involving the extremities.
A. The open procedure is a lesion of the anterior cord.
B. The lesion is made in the anterior spinothalamic tract.
C. The lesion is made in the intermediolateral cell column
D. Results are optimal if the procedure is done bilaterally
E. The percutaneous procedure gives excellent relief of
pain to the C2 segment
- Answer: A
Source: Feler C, Board Review 2005
- The following is true regarding cordectomy operations:
A. An optimally selected patient has normal preoperative
neurologic function
B. It is most useful in pain of the upper extremities
C. It is a commonly performed procedure in patients who
have cancer pain
D. A patient with a preoperative transverse myelopathy is
well selected for the procedure
E. Two of the above
- Answer: D
Source: Feler C, Board Review 2005
- Which of the following statements about major
depression is TRUE?
A. Thirty percent of individuals with a single episode of
major depression develop bipolar disorder
B. The lifetime prevalence rates for adult men range from
3% to 9%.
C. Full recovery from major depression occurs in 25% of
patients by 6 months
D. Relapse after a single episode is about 50%.
E. The average age of onset of unipolar major depression is
50 years
- Answer: D
Explanation:
A. Five to ten percent of individuals with a single episode
of major depression will eventually develop bipolar
disease.
B. The National Comorbidity Survey carried out a
structured psychiatric interview of a representative sample
of the general population and reported a lifetime rate of
major depression of 21.3% in women and 12.7% in men
producing a female-to-male ration of 1.0 to 1.7. A gender
difference was found beginning in early adolescence and
persisting through the mid-50s. Although this increased
tendency for depression in women refl ects a long-term
trend,over the short term, an increase has also been seen in
the rate of depression among young women. The highest
rate occurs in adult women aged more than 44 years
C. 50% of cases of major depression will have full
recovery by 6 months.
D. Major depression is a recurrent illness; the risk of
relapse after one episode is about 50%, whereas it is
greater than 80% after 3 episodes. The average lifetime
number is 4.
E. The average age of onset of unipolar depression is 29
years
Source: Laxmaiah Manchikanti, MD
868. The catheter location for continuous infusion for post-op pain relief for lower abdominal surgery should be A. T2-8 B. T4-L1 C. T10 - L3 D. T12 - L3 E. L1 - L3
- Answer: C
Source: Raj, Pain Review 2nd Edition
- Four days after a left total hip arthroplasty, an obese a
62-year-old woman complains of severe back pain in the
region where the epidural was placed. Over the ensuing
48 hours, the back pain gradually worsens and a severe
aching pain that radiates down the left leg to the knee
develops. The most likely diagnosis is
A. Epidural abscess
B. Epidural hematoma
C. Anterior spinal artery syndrome
D. Arachnoiditis
E. Meralgia paresthetica
- Answer: A
Explanation:
A.Epidural abscess is an exceedingly rare complication of
spinal and epidural anesthesia.
* Symptoms from an epidural abscess may not become
apparent until several days after placement of the block.
* The usual symptoms include severe back pain, sensory
disturbances, and motor weakness.
* Patients with epidural abscesses will complain of
radicular pain approximately 3 days after development of
the back pain.
B. In an epidural hematoma severe back pain is the key
feature.
C. Anterior spinal artery syndrome is characterized
predominantly by motor weakness or paralysis of the
lower extremities.
D. Arachnoiditis starts as a minimal cellular infl ammatory
response.
*It may follow trauma, surgery, tumors, infections,
hemorrhage orsome intrathecal compound administration
* Onset of symptoms varies from hours to months,
resulting in delay in diagnosis
* Symptoms include:
- Radicular pain
- Perineal sensory loss
- Lower extremity paresis or paralysis
* Diagnosis can be made by CT, MRI or myelography
E. Meralgia paresthetica is related to entrapment of the
lateral femoral cutaneous nerve as it courses below the
inguinal ligament and is associated with burning pain over
the lateral aspect of the thigh. It is not a complication of
epidural anesthesia.
- A 49-year-old man presents with painful, recurring
episodes of swelling in his left great toe. He takes 25 mg
of hydrochlorothiazide daily for blood pressure control
but otherwise is in good health. On physical examination,
the patient is afebrile but his great toe is warm, swollen,
erythematous, and exquisitely tender to palpation. He has
several subcutaneous nodules in his pinna. The following
is the most likely diagnosis:
A. Calcium pyrophosphate dihydrate deposition disease
B. Calcium oxalate deposition disease
C. Monosodium urate deposition disease
D. Calcium phosphate deposition disease
E. Osteoarthritis of the great toe
- Answer: C
Explanation:
(Tierney, 42/e, pp 786-790.)
Tophaceous gout is characterized by the fi nding in synovial
fl uid of monosodium urate crystals that are needle-shaped and strongly negative birefringent (bright yellow when
parallel to the axis). Gouty attacks may be precipitated by
trauma, medications that inhibit tubular secretion of uric
acid (aspirin, hydrochlorothiazide), surgery, stress,
alcohol, or a high-protein diet. The patient may have an
accumulation of tophi in and around the joints and
earlobe.
Radiographs may show “rat bite” erosions. Pseudogout is
due to calcium pyrophosphate dihydrate (CPPD)
deposition disease; the crystals here are rhomboid-shaped
and weakly positive birefringent (blue when parallel to the
axis). Calcium oxalate deposition disease is usually seen in
patients with end-stage renal disease; calcium phosphate
deposition disease causes calcifi c tendinitis or Milwaukee
shoulder.
- Your patent is a 38-year-old male who plays in a weekend volleyball league on a regular basis. He has developed posterior shoulder pain that is aching in nature and increases with increased slamming of the ball over the net. You have noticed that his symptoms are provoked
with passive internal rotation and adduction of his arm
behind his back, followed by passive cervical sidebending
to the contralateral side. Which disorder do you suspect?
A. Acromioclavicular arthritis
B. Bennett’s lesion
C. Posterior glenohumeral labral tear
D. Suprascapular nerve entrapment
E. Adhesive Capsulitis
- Answer: D
Source: Sizer Et Al - Pain Practice March & June 2003
- When testing a patients extraocular muscle movements,
you detect that the right eye cannot adduct past the
midline. However, when you move a fi ngertip toward
the patient’s nose, convergence does occur. Which of the
following is the most likely diagnosis?
A. Paralysis of cranial nerve VI
B. Paralysis of cranial nerve III
C. Internuclear ophthalmoplegia
D. Retrobulbar optic neuritis
E. Paralysis of cranial nerve II
- Answer: C
Explanation:
(Goldman, 21/e, p 2240.)
Internuclear ophthalmoplegia (INO) is caused by a lesion
in the medial longitudinal fasciculus (MLF) and may be
due to glioma in children, multiple sclerosis in young
adults, or vascular infarction in the geriatric age group.
INO commonly causes paresis of adduction of the
ipsilateral eye (patients cannot look medially), horizontal
nystagmus in the contralateral abducting eye, and vertical
nystagmus with upward gaze, but convergence is intact.
- According to psychoanalytic theory, which of the
following statements about the development of the
superego is true?
A. It is present at birth
B. It begins to develop during the fi rst two years of life
C. It begins to develop during the fi fth or sixth year of life
D. It begins to develop during puberty
E. It begins to develop in late adolescence
- Answer: C
Explanation:
(Kaplan, pp 206-223.)
Freud maintained that the
superego begins to develop around the age of 5 or 6 as part
of the resolution of the Oedipus complex.At the end of the
phallic stage of psychosexual development (which lasts
from around 211> to 6 years of age), children must
abandon the sexual and aggressive impulses that were
directed toward their parents to avoid the parents’ strong
disapproval. In abandoning these impulses, children
identify with their parents. Part of this identifi cation
involves the internalization of parental standards of
morality; this internalization marks the beginning of the
superego.
Source: Ebert 2004
- At a follow-up visit one month after a 22-year-old male
was newly diagnosed with schizophrenia and started
on chlorpromazine, he has several complaints, listed
below. Which of the following cannot be attributed to
chlorpromazine?
A. Restless feeling
B. Sexual dysfunction
C. Urinary hesitancy
D. Vomiting
E. None of the above
- Answer: D
Explanation:
Antipsychotic agents, particularly prochlorperazine, are
also useful as antiemetic agents, thought to be due to
dopamine blockade at the stomach and at the
chemoreceptor trigger zone of the medulla.
Source: Stern - 2004
875. Which of the following negative emotional states or conditions most commonly precedes relapse in the treatment of addictive behaviors? A. Stress B. Depression C. Anxiety D. Anger E. Frustration
- Answer: C
Explanation:
(Taylor, pp 108-114.)
Negative emotional states of
anxiety, depression, anger, frustration, and stress are
related to relapse in the treatment of addictive behaviors
involved in such disorders as alcoholism, smoking,
obesity, and drug addiction. Most patients (about 70%)
have negative affects preceding the relapse. The most
common negative affect or mood state is anxiety related to
the need for the addicting substance to relieve the anxiety.
This is followed by anger, frustration, and depression.
Furthermore, patients are at increased risk for relapse if
they smoke, drink, eat, and so on in an attempt to reduce
negative affect.
Source: Ebert 2004
- A 40-year-old man is asked to be evaluated by the company
physician because he failed a mandatory random drug
screen. The history indicates a pattern of substance abuse.
The psycosocial factor most likely to be found in this
person is
A. Delusions of grandeur
B. Depression mood
C. Rationalization
D. Denial
E. Antisocial behavior
- Answer: D
Explanation:
(Sierles, pp 295-296. Ebert pp 233-259.)
Even though there is no single personality type associated
with substance abuse, denial is the major psychosocial
factor in these persons. Denial can also complicate the
treatment, even though the substance abuser may admit to
the addiction. Substance abusers do frequently display
delusions of grandeur, are often in a depressed mood,
frequently display antisocial behavior, and often rationalize
their behavior or situation, but denying the seriousness of
their drinking or drug problem and its effect on their life
or loved ones is the most common psychosocial factor
seen. Denial is a form of self-deception that permeates
their psychological and social behavior.
A physician must be constantly aware of the infl uence of
denial on the patient’s self-reported history and the effect
of denial on prognosis.
Source: Ebert 2004
- If resisted shoulder external rotation is the MOST
painful procedure during the shoulder basic functional
examination, which tendon insertion is most likely the
pain generator?
A. Supraspinatus
B. Infraspinatus
C. Subscapularis
D. Deltoid
E. Biceps
- Answer: B
Source: Sizer Et Al - Pain Practice March & June 2003
- What is the approximate scapulo-humeral movement
ratio produced between the glenohumeral joint and
scapulothoracic complex at 40° of arm elevation?
A. A ratio of 1 : 1
B. A ratio of 7 : 1
C. A ratio of 3 : 1
D. A ratio of 5 : 1
E. A ratio of 2 : 1
- Answer: B
Source: Sizer Et Al - Pain Practice March & June 2003
- A 40-year-old male with a diagnosis of moderate to severe
asthma is placed on zileuton. What is the mechanism of
action of zileuton?
A. Inhibition of cytokine production
B. Inhibition of leukotriene production
C. Inhibition of mediator release
D. Inhibition of muscarinic receptor action
E. Inhibition of calcium (Ca2+) channel activity
- Answer: B
- In a patient with spinal stenosis at L5/S1 levels with
history of low back and lower extremity pain, the likely
electrodiagnostic fi ndings are as follows:
A. Reduced amplitude of H-have response
B. Increased amplitude of the somatosensory evoked response
C. Normal F-wave response
D. Reduced conduction velocity of the genitofemoral
nerve
E. Fibrillation in tibialis anterior
- Answer: A
Explanation:
Slowing of nerve conduction velocity occurs through the
region of neurologic impairment. Needle EMG may not
show evidence of membrane instability. Motor unit action
potentials may have increased amplitude and duration
because of collateral innervation, which occurs over time.
Somatosensory evoked potentials will be abnormal to
varying degrees in the dermatomes of the affected nerve
roots. Both F- and H-wave late responses will be
abnormal. Fibrillation or sharp waves in tibialis anterior
are seen with involvement of L4 nerve root.
- You were performing a stellate ganglion block at C6. You withdrew the needle 0.2 cm after inserting the needle
to a depth of 1.4 cm. You were unable to inject due to
resistance and pain. The tip of the needle is most likely
located within the
A. Periosteum
B. Longus colli
C. Vertebral artery
D. Intervertebral disc
E. Subarachnoid space
- Answer: B
882. Epidural use of which of the following opioids would result in the greatest incidence of delayed respiratory depression? A. Sufentanyl B. Fentanyl C. Morphine sulfate D. Hydromorphone E. Meperidine
- Answer: C
Explanation:
Water-soluble drugs such as morphine have a higher
potential for inducing delayed respiratory depression
through cephalad migration in the CNS.
- A 36-year-old male has been experiencing intense pressure to be more productive at work. This has resulted
in his becoming extremely anxious, which makes it very
diffi cult for him to function effectively. He wishes to keep
his job. Physical examination and blood chemistries are
normal. He is given diazepam, which diminishes his
anxiety and allows him to concentrate on his work. What
is the mechanism of action of diazepam?
A. It directly opens the Cl¯ channel of the GABA receptor
B. It increases the frequency of the Cl¯ channel of the GABA
receptor
C. It prolongs the duration of opening of the Cl¯ channel of
the GABA receptor
D. It simulates k receptors
E. It simulates m receptors
- Answer: B
Explanation:
Reference: Hardman, pp 365-367.
Benzodiazepines, such as diazepam, bind to the GABA
receptor/ion channel complex, enhancing GABA-induced
Cl¯ currents related to more frequent bursts of Cl¯
channel opening by GABA.
kappa and μ receptors are opioid receptors.
Source: Stern - 2004
- An elderly patient presents with a complaint of pain in
the distribution of the trigeminal nerve. The patient has
no other medical problems except a history of congestive
heart failure for which he takes digoxin and thiazide.
In addition to his chief complaint, the patient over the
last 72 hours has complained of dysesthesia in the feet,
diffi culty with vision, and emesis on 3 or 4 occasions. The
most appropriate step at this time would be
A. Trigeminal nerve block with bupivacaine
B. Obtain neurologic workup for multiple sclerosis
C. Administration of fentanyl and ondansetron
D. Initiate therapy with carbamazepine
E. Obtain a digoxin level
- Answer: E
Explanation:
The early signs of digitalis toxicity include loss of appetite
and nausea and vomiting. In some patients there may be
pain that is similar to trigeminal neuralgia. Pain or
discomfort in the feet and pain and discomfort in the
extremities may be a feature of digitalis toxicity. Transient
visual disturbances have been reported in patients with
digitalis toxicity.
Source: Hall and Chantigan.
Source: Hall and Chantigan
- An elderly woman presents with persistent and prolonged
thoracic pain after a herpes zoster infection. Which of the
treatments below would be the LEAST effi cacious in the
treatment of her pain?
A. Topical capsaicin ointment
B. Oral clonidine
C. Topical lidocaine patch
D. Oral amitriptyline
E. Transcutaneous electrical nerve stimulation
- Answer: B
Explanation:
Postherpetic neuralgia is defi ned as pain persisting
beyond the healing of the herpes zoster lesions.
The incidence of postherpetic neuralgia increases with
age and occurs in 20% to 50% of patients older than 50
years and greater than 50% in patients older than 80 years.
A, C, D, E. Treatment of established postherpetic neuralgia
has been shown to be resistant to interventions and thus
can be diffi cult. Proven therapies include tricyclic
antidepressants, antoconvulsants, topical local anesthetics,
topical capsaicin, and sympathetic blocks.
B. Oral clonidine, which is used to treat hypertension and
opioid withdrawl, has not been shown to be an effective
treatment for postherpetic neuralgia.
- Complex regional pain syndrome type II (causalgia) is
differentiated from complex regional pain syndrome type
I (refl ex sympathetic dystrophy) by knowledge of its
A. Etiology
B. Rapidity of onset
C. Type of symptoms
D. Affected body region
E. Chronicity
- Answer: A
Explanation:
Complex regional pain syndrome type I (refl ex
sympathetic dystrophy) is a clinical syndrome of
continuous burning pain usually occurring after an
injury or surgery. Patients present with variable
sensory, motor, autonomic, and trophic changes.
Complex regional pain syndrome type II (causalgia)
exhibits the same features of refl ex sympathetic dystrophy,
but the etiology is damage to a major nerve.
- A patient presents with acute onset of pain which started when he was stepping off a curb located over hip and buttock area which is referred to groin and lower
extremity. Physical examination showed no leg length
discrepancy but pain over superior iliac spine. The most
likely diagnosis is:
A. Lumbar facet joint pain
B. Osteoarthritis of hip
C. Lumbar radiculopathy
D. SI joint pain
E. Trochanteric bursitis
- Answer: D
- In traditional psychoanalysts, transference is the process wherein:
A. Psychic energy, or libido, is transferred from the id to the
ego and superego
B. A patient invests the analyst with attitudes and feelings
derived from vital earlier associations
C. Certain psychological symptoms seemingly defer to
new symptoms that frequently are more accessible to
analysis
D. Early object choices are gradually decathected
E. Latent dream content is transformed into manifest content
- Answer: B
Explanation:
(Kaplan, pp 885-888.)
In traditional psychoanalytic treatment, analysts purposely
reveal very little about themselves to their patients. That is
intended to help promote transference-to create an
ambiance that facilitates a patient’s ability to transfer his or
her past emotional attachments to the psychoanalyst. The
analyst becomes a substitute for the parental fi gure. In
positive transference, the patient becomes attached to the
analyst to obtain love and emotional satisfaction, where as
in negative transference the analyst is seen as an unfair,
unloving, and rejecting parental fi gure. Interpretations of
transference may help the patient see the positive or
negative feelings as a refl ection of previous of emotional
entanglements
Source: Ebert 2004
- The therapeutic action of b-adrenergic receptor blockers
such as propranolol in angina pectoris is believed to be
primarily the result of
A. Reduced production of catecholamines
B. Dilation of the coronary vasculature
C. Decreased requirement for myocardial oxygen
D. Increased peripheral resistance
E. Increased sensitivity to catecholamines
- Answer: C
Explanation:
Beta-adrenergic receptor blockers cause a slowing of heart
rate, lower blood pressure, and lessened cardiac
contractility without reducing cardiac output. There is also
a buffering action against adrenergic stimulation of the
cardiac autoregulatory mechanism. These hemodynamic
actions decrease the requirement of the heart for oxygen.
Source: Hardman, pp 855-856
- True statements with worker’s compensation coverage are as follows:
A. State-mandated worker’s compensation programs also
cover all types of federal employees.
B. Diffi cult cases are automatically settled after 12 months.
C. Self-insured employers that do not subscribe to state
laws are foolproof from litigation
D. Self-insured employers that subscribe to state laws and
administer their own benefi ts are very rigid and do not
accommodate injured workers at light duty positions.
E. Inherent problems with worker’s compensation system
include poor understanding of the cause of pain, particularly
in the absence of defi nitive diagnostic tests
resulting in unsuccessful return to work and ineffective
case management, etc.
- Answer: E
Source: Cole and Hearring to the evaluation of Permanent
Impairment, 2001.
- A 39-year-old man presents with progressive weakness of his arms and legs. He noticed diffi culty in performing tasks such as buttoning up his shirt several months ago, and his symptoms have continued to worsen. On physical examination, cranial nerve and sensory findings are normal. Severe atrophy and fasciculations are seen in the legs, arms, and tongue. The patient has a spastic
muscle tone, hyperactive refl exes, and bilateral extensor
plantar refl exes. Which of the following is the most likely
diagnosis?
A. Werdnig-Hoffmann disease
B. Multiple sclerosis
C. Pott’s disease
D. Amyotrophic lateral sclerosis
E. Todd’s paralysis
- Answer: D
Explanation:
(Tierney, 42/e, pp 990-991.)
Amyotrophic lateral sclerosis (ALS) is a degenerative
disease that is the result of lower (anterior horn cells) and
upper (corticospinal tracts) motor neuron loss. Patients
present with asymmetric muscle weakness, atrophy,
fasciculations, spasticity, hyperactive refl exes, and extensor
plantar refl exes. Patients may complain of dysphagia and
diffi culty holding the head up. Pott’s disease tuberculosis
of the thoracic vertebral bodies. Todd’s paralysis is a
transient paralysis following a seizure. Werdnig-Hoffmann disease is fl oppy baby disease; infants present
with fasciculations. Poliomyelitis is a 1m motor neuron
disease.
892. The following contrast in recommended in interventional techniques A. Hypaque B. Renographin C. Non-ionized water soluble contrast D. Ionized water soluble contrast E. Ionized non water soluble contrast
- Answer: C
Source: Racz G. Board Review 2003
- A 34-year-old man has been diagnosed with chronic
paranoid schizophrenia for 10 years. He is currently
in a psychiatric hospital and is not on psychotropic
medications. More than 50% of individuals with this
diagnosis and off medications would have abnormalities
in which of these tests?
A. Lactate infusion test
B. Dexamethasone suppression test
C. Eye pursuit test
D. Thyrotropic releasing hormone (TRH) stimulation test
E. Prolactin stimulation test
- Answer: C
Explanation:
(Sierles, pp 185-187. Ebert, pp 268-270.)
· Many psychiatric disorders manifest evidence of brain
dysfunction.
· Evidence of brain dysfunction has been found in 50% or
more of patients with schizophrenia. For example, a
neurologic examination will demonstrate soft signs (e.g.,
grasp refl ex, rooting refl ex, motor impersistence) in 70%
of adult schizophrenics; 75% (whether ill or recovered)
will also demonstrate abnormalities in eye pursuit; 75%
will have moderate-to-severe bilateral impairment on
neuropsychological tests; and 50% will have nonspecifi c
abnormalities on the EEG. Also, 50% will have some
cortical atrophy and ventricular enlargement on imaging
tests.
· The lactate infusion test induces panic behavior in 80%
of patients with panic disorder, but not in patients with
schizophrenia.
Source: Ebert 2004
- A 23-year-old woman complains of periodic, throbbing,
right-sided headaches accompanied by nausea and
vomiting. On physical examination during the time of
headache, the patient demonstrates a right oculomotor
nerve palsy. MRI is normal. Choose correct type of
headache:
A. Complicated migraine
B. Basilar migraine
C. Classic migraine
D. Common migraine
E. Temporal arteritis
- Answer: A
Explanation:
(Tierney, 42/e, pp 947-949)
A. Complicated migraines may be preceded by aura and
are headaches accompanied by sensory or motor defi cits
or muscle palsies.
The patient described is having a specifi c kind of
complicated migraine called an ophthalmoplegic migraine.
A mnemonic for migraine is POUND (Pulsatile, lasts
One day, Unilateral, Nausea, and interferes with Daily
activities).
B. Basilar artery migraine is a variant of classic migraine
in which the aura consists of drop attacks, confusion,
blindness, and vertigo (all signs of basilar artery
ischemia).
C. Classic migraine is a unilateral headache that is
pulsatile and throbbing in nature and is preceded by a
prodromal aura consisting of scotomas (black spots),
scintillations (light fl ashes), or hemianopsia.
D. Common migraines lack a prodromal aura.
E. Patients with temporal arteritis are older (>50 years
old) and have headaches along with jaw claudication and
tenderness over the temporal artery.
- A patient has had an implanted intrathecal infusion
pump for post-laminectomy syndrome for the past 3
years. He has had relatively good pain control with a
combination infusion of morphine, bupivacaine, and
baclofen. You are asked to evaluate him in the emergency
room for increasing low back pain associated with new
onset of right leg pain and right leg weakness. Physical
examination reveals positive right straight leg raising
with loss of right Achilles refl ex. Plain x-ray has identifi ed
the titanium catheter tip marker at the T8 level in the
ventral intrathecal space. The following statement is
false:
A. Most patients who are diagnosed with catheter tip granuloma
present with gradual loss of pain control associated
with gradual onset of lower extremity neurological
defi cits evolving over weeks and months.
B. When catheter tip granuloma is diagnosed, surgical
removal of the catheter and pump is the treatment of
choice.
C. MRI with and without gadolinium enhancement is the
imaging study of choice to assess catheter tip granuloma.
D. Baclofen when used alone in the pump for spasticity
management has not been implicated in catheter tip
granuloma.
E. Right lumbar radiculopathy is a much likelier diagnosis
than catheter tip granuloma in this patient
- Answer: B
Explanation:
Reference:
Management of Intrathecal Catheter-Tip Infl ammatory
Masses: A Consensus Statement
Hassenbusch et. Al. Pain Medicine 2002
Infl ammatory mass formation at the tip of an implanted
intrathecal catheter is a rare but potentially devastating
complication of intrathecal drug infusion. Hassenbush et.
Al. reviewed published and unpublished case reports and
their own experiences to recommend methods to diagnose
and treat catheter-tip infl ammatory masses in the above
article.
After comprehensive review, the Hassenbush consensus
panel concluded that:
Fluctuations in patients’ subjective symptoms and
underlying pain levels are common after the implantation
of drug delivery systems, but the occurrence of new or
extraordinary complaints that require unexpected
analgesic dose changes should alert physicians to consider
a catheter-tip mass among other possibilities in the
differential diagnosis. Gradual, insidious neurological
deterioration weeks or months after the appearance of
subjective symptoms was the most common clinical
course for catheter tip granulomas before the onset of
myelopathy or cauda equina syndrome in cases reported to
date.
Physicians should have a low threshold for performing
an imaging study to confi rm or rule out the presence of a
catheter-tip mass in patients with suspicious symptoms or
physical fi ndings. Unless medically contraindicated, MRI
with and without intravenous gadolinium contrast
enhancement is the imaging procedure of choice. CT
myelogram is an acceptable alternative and is equally
sensitive and reliable. Catheter-tip masses are visualized
best on intravenous contrast-enhanced T2-weighted
images. The mass appears as an enhancing lesion having
the tip of the drug administration catheter embedded
within it.
Not all patients with catheter tip granuloma require
catheter and pump removal. When catheter tip granuloma
is diagnosed, optimal management should take into
account the patient’s clinical condition, the wishes of the
patient and the available options for chronic pain
management. Mildly symptomatic patients with small
masses that are diagnosed during investigation of
diminished analgesic effi cacy or other subjective
complaints have been managed safely and successfully
without open surgical decompression or removal of the
mass. These masses did not signifi cantly compress neural
structures, nor compromise neurological function, and
were treated with prompt discontinuation of intrathecal
drug administration. Shrinkage or disappearance of the
mass was documented on follow-up imaging studies after
an interval of 2-5 months. Consequently, catheter-tip
infl ammatory masses that are detected early in the clinical
course can be treated safely and effectively by maneuvers
directed at modifying rather than removing the drug
infusion system. If the decision is made to leave the
infusion system in place, the responsible physician
eventually must decide whether to continue intrathecal
therapy and whether to change the dose, concentration, or
the drug(s) being infused. Alternatives to complete
removal of the catheter and pump include ceasing or
changing drug infusion and:
1.Repositioning of the catheter at a different spinal level.
2.Placing a new catheter to replace the existing catheter.
3.Allowing the catheter and pump to remain dormant for
a period of time.
In contrast, patients presenting with paraplegia or
progressive myelopathy or with apparently fi xed
neurological defi cits of short duration may require
emergent operative intervention because of concern that
delayed treatment could foreclose the possibility of
neurological recovery. Surgical intervention to remove the
mass and/or de-compress the spinal canal has restored
neurological function or prevented further neurological
deterioration in several reported cases. The extent of
resection was limited in some cases owing to adhesions to
the spinal cord or nerve roots or because of the ventral
location of a mass beneath the thoracic spinal cord.
Because the masses were not neoplastic, in several cases the
postoperative residual mass gradually shrank or
disappeared over time.
Source: Schultz D, Board Review 2004
- A 70-year old patient presents with a history of increasing
pain in the back, buttocks and leg. Pain in the leg worsens
with standing and walking. Pain is relieved on bending
forward. No neurological defi cits were identifi ed on
physical examination. Acetaminophen gives minimal
relief. Your next treatment would be administration of:
A. Opioids
B. Epidural steroid injections
C. Non-steroidal anti-infl ammatory drugs
D. Facet joint injections
E. Transcutaneous electrical stimulation
- Answer: C
Explanation:
The pain noted by this patient may include
musculoskeletal pain associated with spondylosis, as well
as radicular pain or neurogenic claudication.The fi rst
approach should be noninvasive, and NSAIDs are
appropriate, because acetaminophen was minimally
- A 60-year-old man was involved in a motor vehicle
accident and suffered multiple long bone fractures and a
severe injury to the pelvis. Two days following admission
to the hospital, he develops fever, tachypnea, and
tachycardia. The rest of his physical examination reveals
chest, neck, and conjunctival petechiae. Respiratory exam
reveals scattered crackles bilaterally but no wheezes.
Pulse oximetry reveals a hemoglobin saturation of 80%
on room air. Which of the following is the most likely
diagnosis?
A. Pneumothorax
B. Pneumonia
C. Exacerbation of chronic obstructive pulmonary disease
(COPD)
D. Anemia from traumatic blood loss
E. Fat embolism syndrome
- Answer: E
Explanation:
(Goldman, 21/e, p 448.)
The signs and symptoms of fat embolism syndrome are
those of adult respiratory distress syndrome (ARDS) in
association with musculoskeletal trauma. It usually occurs
2 to 4 days after the injury. The predominant feature is
respiratory failure. Petechiae are found in 50 to 60% of
patients, generally on the anterior chest and neck, axillae,
and conjunctiva.Although fractures of the pelvis may
cause life-threatening blood loss and subsequent
hypovolemic shock, the patient will probably have other symptoms, such as oliguria, hypotension, pale conjunctiva,
clouded sensorium, and cool extremities.
- Following cholecystectomy, a patient is receiving
bupivacaine by intrapleural infusion at 8mL/hr. The
patient is noted to have a Horner’s syndrome and
inadequate pain relief. The next step in managing this
patient is to :
A. Increase the rate of bupivacaine infusion
B. Remove and reinsert the catheter
C. Obtain neurology consultation
D. Obtain an MRI of the head
E. Perform a chest radiograph
- Answer: B
Explanation:
Assuming that you wish to continue the postoperative
intrapleural infusion, the catheter should be removed and
replaced. A Horner’s syndrome is a recognized side effect
of interpleural infusions. Poor pain relief means that the
catheter not covering the area of the incision. The
Horner’s syndrome means that the infusion is not
intravascular
- Patients with a strong sense of an external locus of
control of health will be more apt to respond to inpatient
treatment in the following way:
A. They delegate control of their health to their doctor or
signifi cant other
B. They can be relied on to follow treatment orders when
they are discharged for outpatient follow-up
C. They respond poorly and less comfortably to inpatient
care
D. They prefer to make as many decisions about their care
as possible
E. They prefer to maximize their own decision making
about their own health care
- Answer: A
Explanation:
(Sierles, pp 103-104. Wedding, pp 378-390.)
A. Persons with a strong sense of an external locus of
health control delegate responsibility for their health to an
external force, such as fate, powerful others, chance, or
God.
B. As outpatients they cannot be relied on to take
responsibility for their own care.
C. They can be relied on to follow treatment orders in the
hospital, where they are in an authoritarian system.
D. They prefer to make as few decisions as possible about
their own health and prefer to accept the authority and
orders of their own physician, except if the authority is not
present to follow up on them.
E. Patients with a strong sense of internal control tend to
accept responsibility for and control their own health.
This information can be of help to physicians by allowing
them to establish follow-up procedures that will ensure
maximum compliance.
Source: Ebert 2004
- Patients with low back pain have been found to have:
A. Normal levels of aerobic fi tness compared to normal
controls.
B. An inability to improve their aerobic capacity.
C. Protection against low back pain at work after a period
of aerobic training.
D. Spine problems that would prohibit most forms of aerobic
exercise.
E. Lumbar disc herniation in 90% of cases
- Answer: C
Source: Malanga G, Board Review 2003
901. Patient notes the “worst headache ever” after exercise. Physical examination shows neck pain with movement. The next course of action is: A. MRI of head B. MRI of neck C. CT of head D. CT of neck E. Cervical spine films
- Answer: C
Explanation:
the patient needs a head CT to evaluate for possible
subarachnoid hemorrhage.The subsequent step would be
a spinal tap, assuming no mass lesion or shift.
- A patient presents with acute low back and lower
extremity pain. Motor examination showed weakness
with foot inversion. There was sensory defi cit on the
medial aspect of the leg. The most likely diagnosis is:
A. L3/4 disc herniation with L4 nerve root involvement
B. L5 nerve root involvement with L4/5 disc herniation
C. S1 nerve root involvement with L5/S1 disc herniation
D. L3 nerve root involvement with L2/3 disc herniation
E. L4/5 disc herniation with S1 nerve root involvement
- Answer: A
Explanation:
L4 nerve root involvement with L3/4 disc herniation
shows weakness of tibialis anterior demonstrated by
weakness of foot inversion. Refl exes are patellar and
sensation is on the medial leg.
Source: Hoppenfeld S. Orthopaedic Neurology. A
Diagnostic Guide to Neurologic Levels. Philadelphia,
LWW, 1997.
903. Which of the following is not an indication for stretching? A. Prolonged immobilization B. Restricted mobilitiy C. Connective tissue diseases D. Structural damage due to trauma E. Recent fracture
- Answer: E
Explanation:
(Raj, Practical Mgmt of Pain, 3rd Ed., page 535-536)
Indications
Prolonged immobilization leading to adhesions and
contractures.
Restricted mobility.
Connective tissue or neuromuscular diseases.
Structural damage secondary to trauma.
Congenital or acquired bony deformities.
Contraindications
Restricted motion secondary to a bony block.
After a recent fracture.
Evidence of an acute infl ammatory or infective process,
either in or around a joint.
Patients in whom contractures are the chief means of
providing joint stability
Source: Shah RV, Board Review 2005
- A 46-year-old woman has a l-month history of headache.
She has no past medical history of headache and no
family history of headache. She does not use illicit drugs,
drink alcohol, or smoke cigarettes. Physical examination
reveals alexia, agraphia, acalculia, right-left confusion,
and linger agnosia. An MRI of the brain with gadolinium
is most likely to show which of the following?
A. Frontal lobe lesion
B. Parietal lobe lesion
C. Temporal lobe lesion
D. Occipital lobe lesion
E. Cerebellar lesion
- Answer: B
Explanation:
(Tierney, 42/e, p 969-971.)
MRI will most likely reveal a lesion of the parietal lobe.
Parietal lobe lesions may produce contralateral
hyperpathia and pain (thalamic syndrome) and
Gerstmann syndrome (alexia, agraphia, acalculia, right-left
confusion, and fi nger agnosia).
Occipital lobe lesions produce partial fi eld defects.
Temporal lobe lesions produce seizures, lip smacking,
olfactory or gustatory hallucinations, and behavioral
changes. Frontal lobe lesions lead to intellectual decline
and personality changes. The most common adult primary
tumors are gliomas.
- A college student presents with complain of pain in
fi ngers with blanching and cyanosis of her fi ngertips in
cold weather and numbness. She has a 6-month history
of dysphagia and arthralgias. She does not smoke or
take any medications. On physical examination, the
skin of her hands appears to be taut and atrophic with a
fl exion deformity from the tight skin (sclerodactyly). The
following is the most likely diagnosis:
A. Rheumatoid arthritis
B. Progressive systemic sclerosis
C. Dermatomyositis
D. Ulcerative colitis
E. Sarcoidosis
- Answer: B
Explanation:
(Tierney, 42/e, pp 813-814.)
The patient presents with symptoms suggestive of
scleoderma or progressive systemic sclerosis (PSS). This
disease, when diffuse, involves the skin, joints, lungs,
heart, and gastrointestinal system. Limited systemic
sclerosis (lSSc) was formerly known as the CREST
syndrome (Calcinosis cutis, Raynaud’s phenomenon,
Esophageal dysfunction, Sclerodactyly, and
Telangiectasia).
Raynaud’s phenomenon may be associated with tobacco
use, medication use (ß-adrenergic blockers), or diseases
such as systemic lupus erythematosus, rheumatoid
arthritis, carpal tunnel syndrome, or thromboangiitis
obliterans.
Dermatomyositis is a systemic disease characterized by a
violaceous rash of the eyelids and periorbital areas
(heliotrope) and fl at, violaceous papules over the knuckles
(Gottron sign). The rash seen in ulcerative colitis is
pyoderma gangrenosum. These painful ulcers are large and
irregular and drain purulent, hemorrhagic exudates.
Sarcoidosis is a systemic disease with skin
manifestations, bilateral hilar adenopathy, and pulmonary
disease. Patients with sarcoidosis may present with
erythema nodosum, which typically takes the form of
multiple fi rm, red, painful plaques that are bilateral and
most frequently distributed on the legs. Musculoskeletal
fi ndings in sarcoidosis include arthritis and tenosynovitis
906. What is transmitted in the tarsal tunnel? A. Anterior tibial tendon B. Posterior tibial nerve C. Flexor hallucis longus tendon D. Posterior tibial tendon E. Flexor digitorum longus tendon
- Answer: A
Explanation:
The tarsal tunnel is bounded by a fl exor retinaculum that
spans the medial malleolus and the calcaneus
The tibialis posterior, fl exor digitorum longus, and fl exor
hallucis longus tendons and the posterior tibial artery and
nerve pass through the tarsal tunnel
Source: Shah RV, Board Review 2004
- A 30-year-old obese woman presents with a 2-month
history of a nonthrobbing headache that is constant and
dull in nature. The headache is worsened with bending
over or sneezing and on awakening in the morning. The
patient also complains of blurred vision and occasional
diplopia. Funduscopic examination reveals blurring of
the optic discs bilaterally and no other neurologic defi cit.
Which of the following is the most likely diagnosis?
A. Infratentorial brain tumor
B. Pseudo tumor cerebri
C. Supratentorial brain tumor
D. Pituitary adenoma
E. Metastatic brain tumor
- Answer: B
Explanation:
(Tierney, 421e, p 974.)
Patients with pseudotumor cerebri (benign intracranial
hypertension) present with headache and papilledema.
They are often obese women in their childbearing years.
Other possible causes include hypervitaminosis A and the
use of oral contraceptives or antibiotics (tetracycline).
Lumbar puncture will reveal an elevated opening pressure.
Treatment includes weight reduction and repeated lumbar
punctures to reduce intracranial pressure. A complication
of pseudo tumor cerebri is blindness; patients with visual
changes may require emergency optic nerve sheath
decompression.Pituitary adenomas are benign tumors that
may cause a bitemporal hemianopsia and endocrine
disturbances, such as hyperprolactinemia (galactorrhea),
acromegaly or gigantism, and Cushing’s disease. A
ruptured berry aneurysm causes a subarachnoid
hemorrhage (SAH). Patients present with the acute onset
of severe headache, photophobia,and neck stiffness.Adults
commonly have supratentorial primary brain tumors
(astrocytoma including glioblastoma multiforme is the
most common), while children have infra tentorial
primary brain tumors (medulloblastoma is the most
common). Overall, metastatic brain tumors are more
common than primary brain tumors. The most common
metastatic brain tumors come from the Lung, Breast, Skin,
Kidney, or GI tract (mnemonic: Lots of Bad Stuff KillsGlia). The headache of tumor is often continuous;
exacerbated by coughing, sneezing, movement, or the
Valsalva maneuver; and worse in the morning.
- A 24-year-old woman has a 2-year history of recurrent
right-sided headaches that are throbbing in nature and
are preceded by 30 min of scintillating scotomas and
fortifi cations. Choose correct type of headache:
A. Complicated migraine
B. Basilar migraine
C. Classic migraine
D. Common migraine
E. Temporal arteritis
- Answer: C
Explanation:
(Tierney, 42/e, pp 947-949)
A.Complicated migraines may be preceded by aura and are
headaches accompanied by sensory or motor defi cits or
muscle palsies.
The patient described is having a specifi c kind of
complicated migraine called an ophthalmoplegic migraine.
A mnemonic for migraine is POUND (Pulsatile, lasts
One day, Unilateral, Nausea, and interferes with Daily
activities).
B. Basilar artery migraine is a variant of classic migraine
in which the aura consists of drop attacks, confusion,
blindness, and vertigo (all signs of basilar artery
ischemia).
C. Classic migraine is a unilateral headache that is
pulsatile and throbbing in nature and is preceded by a
prodromal aura consisting of scotomas (black spots),
scintillations (light fl ashes), or hemianopsia.
D. Common migraines lack a prodromal aura.
E. Patients with temporal arteritis are older (>50 years
old) and have headaches along with jaw claudication and
tenderness over the temporal artery.
909. A long-distance runner develops foot pain with exercise. CHOOSE CORRECT DIAGNOSIS: A. Hammer toe B. March fracture C. Genu valgum D. Genu varum E. Bunion
- Answer: B
Explanation:
(Seidel, 5/e, p 732.)
Improper footwear results in lateral deviations of the great
toe, extensor, and fl exor hallucis longus tendons (bunion
formation). Hammer toe often affects the second toe. The
metatarsophalangeal joint is dorsifl exed and the proximal
interphalangeal joint displays plantar fl exion. A stress
fracture of a metatarsal is called a march fracture. Stress
fractures result in bone resorption followed by insuffi cient
remodeling due to continued activity Stress fractures
occur in the tibia as well as the metatarsal; examination
typically reveals point tenderness and swelling. In genu
varum (bowleg), the lateral femoral condyles are widely
separated when the feet are placed together in the
extended position. In genu recurvatum, the knee
hyperextends, and in genu impressum, there is fl attening
and bending of the knee to one side with displacement of
the patella. Pes planus is a fl attened longitudinal arch of
the foot, often called fl at foot. Morton’s neuroma causes
pain in the forefoot that radiates to one or two toes with
tenderness between the two metatarsals. The pain may be
further aggravated by squeezing the metatarsals together.
- The mechanism of cryotherapy’s affect on pain occurs
by:
A. Increasing metabolic rate of tissues.
B. Vasodilatation of blood vessels.
C. As a counter-irritant.
D. Improving contractility of muscle and ligament fi bers.
E. Increasing nerve conduction along pain pathways.
- Answer: C
Source: Malanga G, Board Review 2003
911. An aphasia is most likely to be associated with a lesion of A. The hippocampus B. The temporal lobe C. The parietal lobe D. The limbic system E. The reticular activating system
- Answer: B
Explanation:
(Guyton, pp 669-671.)
Aphasia is a language disorder in which a person is unable
to properly express or understand certain aspects of
written or spoken language. It is caused by lesions to the
language centers of the brain, which, for the majority of
persons, are located within the left hemisphere in the
portions of the temporal and frontal lobes known as
Wernicke’s and Broca’s areas, respectively. Language
disorders caused by memory loss, which could be the
result of a hippocampal lesion, are not classifi ed as
aphasias.
- McKenzie exercises:
A. Would include repetitive extension even if radicular
symptoms are increased.
B. Stresses “centralization” of back pain symptoms.
C. Is most helpful in chronic nonorganic low back pain.
D. Is contra-indicated in acute disc herniations.
E. Would stress on fl exion exercises
- Answer: B
Source: Malanga G, Board Review 2003
- Parrafi n Wax is most helpful in which of the following
conditions:
A. Following an acute burn injury.
B. Joint pain of the hands from osteoarthritis.
C. Carpal tunnel syndrome.
D. An acute hand fracture.
E. Ankle Strain
- Answer: B
Source: Malanga G, Board Review 2003
- The most important role of the gamma motoneurons is
to
A. Stimulate skeletal muscle fi bers to contract
B. Maintain afferent activity during contraction of muscle
C. Generate activity in Ib afferent fi bers
D. Detect the length of resting skeletal muscle
E. Prevent muscles from producing too much force
- Answer: B
Explanation:
(Berne, 3/e, pp 117-118.)
The gamma motoneurons innervate the intrafusal fi bers of
the muscle spindles. When a skeletal muscle contracts, the
intrafusal muscle fi ber becomes slack and the Ia afferents
stop fi ring. By stimulating the intrafusal muscle fi bers
during a contraction,the gamma motoneurons prevent the
intrafusal muscle fi bers from becoming slack and thus
maintain fi ring during the contraction
915. A 55-year-old woman walks by lifting one foot further off the ground than the other. Choose correct description of gait: A. Ataxic gait B. Parkinsonian gait C. Spastic hemiplegic gait D. Steppage gait E. Scissor gait
- Answer: D
Explanation:
(Berne, 3/e, pp 117-118.)
The gamma motoneurons innervate the intrafusal fi bers of
the muscle spindles. When a skeletal muscle contracts, the
intrafusal muscle fi ber becomes slack and the Ia afferents
stop fi ring. By stimulating the intrafusal muscle fi bers
during a contraction,the gamma motoneurons prevent the
intrafusal muscle fi bers from becoming slack and thus
maintain fi ring during the contraction
- A Middle aged woman presents with a one year history
of pain and morning stiffness accompanied by swelling
of her wrists and the proximal interphalangeal joints of
both hands. She also has knee pain and swelling of knee
joints. Physical examination reveals synovial tenderness
and swelling of her knees, wrists, and proximal
interphalangeal joints. She has subcutaneous nodules
in the extensor area of her right forearm.The right knee
has a positive bulge sign consistent with an effusion. The
most likely diagnosis is:
A. Osteoarthritis
B. Rheumatoid arthritis
C. Septic arthritis
D. Chondrocalcinosis
E. Scleroderma
- Answer: B
Explanation:
(Tierney, 42/e, pp 829-831.)
C. A septic joint will usually produce systemic symptoms
such as fever.
A. Osteoarthritis produces a short period of morning
stiffness and often affects the distal interphalangeal joints.
D. Chondrocalcinosis is a radiologic fi nding (destructive
arthropathy) associated with pseudogout or CPPD
crystals.
B. The patient most likely has rheumatoid arthritis since
she meets four of the seven criteria as classifi ed by the
American College of Rheumatology:
Symmetric polyarthritis for over 3 months
Morning stiffness lasting more than 1 h
Rheumatoid nodules
Arthritis of more than three joint areas
Involvement of the joints of the hands and wrists;
patients may have swan-neck deformity
(hyperextension of the proximal interphalangeal joints
with compensatory fl exion of the distal joint),
boutonniere deformity (extension of the distal
interphalangeal joint), or ulnar deviation of the digits
A positive rheumatoid factor (RF)
Erosions or decalcifi cation on radiographs
917. All of the following psychiatric disorders are diagnosed more often in women than in men. The most frequently diagnosed disorder in women is A. Depression B. Obsessive-compulsive disorders C. Anxiety disorders D. Bulimia E. Anorexia nervosa
- Answer: A
Explanation:
(Fauci, pp 21-24.)
The most frequently diagnosed
psychological disorders in women are depression, anxiety
disorders, bulimia, and anorexia nervosa. Obsessivecompulsive
disorders are almost equally distributed
between adult men and women(prevalence about 2%), but
with a slightly higher prevalence among boys than girls.
Psychological disorders may have a higher prevalence in
women because men are more reluctant to consult a
physician for emotional problems. Another explanation is
that physicians may be more apt to diagnose vague mood
and anxiety complaints as psychological if there is no
obvious organic basis.
Source: Ebert 2004
- A 31-year-old man complains of daily throbbing
headaches for the last 2 weeks. He has approximately eight
episodes per day, each lasting 20 min. The headaches are
localized to the left periorbital area and are accompanied
by tearing of the left eye, left ptosis, rhinorrhea, and left
facial redness. The patient remembers having a similar
problem 2 years ago that lasted for 3 weeks. He did not
seek medical help at that time. The patient feels that the
headaches are often precipitated by drinking a glass of
wine. Which of the following is the most likely diagnosis?
A. Migraine headache
B. Cluster headache
C. Tension headache
D. Trigeminal neuralgia
E. Sinusitis
- Answer: B
Explanation:
(Tierney, 42/e, pp 948-949.)
Cluster headaches are often referred to as “suicide
headaches” because of the severity of the symptoms. These
recurring headaches are accompanied by facial fl ushing,
nasal stuffi ness, tearing, and a partial Horner syndrome
(there is no anhidrosis). They are more common in men
(the usual age is 20 to 50)than women and are exacerbated
by alcohol use.Migraine headaches do not have this timing
or duration. Tension headaches are bilateral,non throbbing, and symmetric. They are usually located in
the frontal or occipital areas of the skull and are thought to
be related to muscle contraction. They are often described
as being viselike. The headache of sinusitis is not abrupt in
onset or cessation, and patients often have tenderness with
percussion of the sinuses. Trigeminal neuralgia (tic
douloureux) is a paroxysmal severe facial pain over the
distribution of the trigeminal nerve. Women are affected
more than men, and patients are usually over the age of 40.
The pain of trigeminal neuralgia can be triggered by
simply touching the skin near the nostril.
919. For diagnostic lumbar sympathetic block commonest sites include: A. L1-L2 B. L2-L3 C. L3-L4 D. L4-L5 E. L5-S1
- Answer: B
Source: Racz G. Board Review 2003
- Migraine symptoms are most likely due to:
A. Vasoconstriction.
B. Epileptiform discharges.
C. Cerebral edema.
D. Decreased cerebral metabolism due to spreading cortical depression.
E. Vasodilatation
- Answer: D
Explanation:
Studies of migraine have focused on vascular factors
indicating that vasoconstrictive drugs reduce the
amplitude of pulsation in the superfi cial temporal artery
but that this does not always reduce headache.It is believed
that extracranial vasodilatation is the cause of headache
and intracranial vasoconstriction is the cause of
neurological symptoms. Currently, the concept that
“spreading cortical depression,” which is a primary neural
(not vascular) event, is the major migraine mechanism.
This cortical depression leads to hypometabolic state and
hypoperfusion. The role of unstable serotonergic
neurotransmission in this cortical depression in migraine
is being explored. (Neurology 43 [suppl. 3], p. 51, 1993;
Journal of Neurophysiology 7, pp. 359-390, 1941;
Source: Neurology for the Psychiatry specialty Board
Review By Leon A. Weisberg, MD
- A 35-year old man presented with constant low back pain
that radiated to the left or right upper buttock region
with occasional radiation to the thigh and calf posteriorly
with tingling sensation in the left heel. The symptoms
started approximately a year ago when he lifted a heavy
box which caused the gradual onset of low back pain at
the time with increasing intensity in a week. His motor
examination was grossly within normal limits. However,
he had a positive left straight leg raising at 50°. There was
decreased sensation to pin prick on the lateral side of the
foot on the left side. The following MRI shows:
A. L4/5 disc herniation
B. L5/S1 disc herniation
C. Large osteophyte pressing on L5 nerve root
D. Large osteophyte pressing on L4 nerve root
E. Facet joint arthritis causing spinal stenosis
- Answer: B
Explanation:
Axial T2-weighted MRI scan at the lumbosacral level.
The arrow shows the degree of disc protrusion and the
effect that it is having on the pain sensitive anterior part of
the dural tube (D) and, to some extent, on the S1 nerve
roots (small white arrows). R = right side of patient. The
rectangle shows the approximate area shown in C.
Lateral T2 weighted MRI scan showing the lumbosacral
spine. S1 = fi rst sacral segment. The posterior disc
protrusion at the L5/S1 level is shown by the black arrow;
it can be seen compressing the anterior part of the dural
tube (D) (thecal sac). Note that the disc is becoming
‘black’ between L5 and S1 which indicates that it is
undergoing dehydration (desiccation) as a result of injury.
The L4/5 disc shows some early desiccation with
essentially normal disc hydration at the levels above.
A 200-micron thick histological section from a cadaver
with a similar but less extensive, disc protrusion; this is to
orientate the reader to the various anatomical structures.
The histological section is represented approximately by
the area within the rectangle on (D). R = right nerve roots
budding off from the dural tube (D) containing small nerve roots from the cauda equina (C). H = hyaline
cartilage on the zygapophysial joint facet surfaces. L =
ligamentum fl avum; N = spinal nerve; S = spinous process.
Open arrow head = intervertebral disc protrusion.
Source: Giles LGF. 50 Challenging Spinal Pain Syndrome
Cases. Edinburgh, Butterworth Heinemann, 2003.
- Sumatriptan succinate is effective for the treatment of
acute migraine headaches by acting as
A. An antagonist at BETA1 – and BETA2 – adrenergic receptors
B. A selective antagonist at histamine (H1) receptors
C. An inhibitor of prostacyclin synthase
D. An agonist at nicotinic receptors
E. A selective agonist at 5-hydroxytryptamine 1D (5-HT1D)
receptors
- Answer: E
Explanation:
Sumatriptan is closely related to serotonin (5-HT) in
structure, and it is believed that the drug is effective in the
treatment of acute migraine headaches by virtue of its
selective agonistic activity at 5-HT1D receptors. These
receptors, present on cerebral and meningeal arteries,
mediate vasoconstriction induced by 5-HT. In addition, 5-
HT1D receptors are found on presynaptic nerve terminals
and function to inhibit the release of neuropeptides and
other neurotransmitters.It has been suggested that the
pain of migraine headaches is caused by vasodilation of
intracranial blood vessels and stimulation of
trigeminovascular axons, which cause pain and release
vasoactive neuropeptides to produce neurogenic
infl ammation and edema. Sumatriptan acts to reduce
vasodilation and the release of neurotransmitters and,
therefore, reduces the pain that is associated with migraine
headaches. Oher antimigraine drugs (e.g.,ergotamine and
dihydroergotamine) also exhibit high affi nities for the 5-
HT1D-receptor site
Source: Katzung, pp 280-281.
923. What of the following is not an indication for a glossopharyngeal nerve block? A. Glossopharyngeal neuralgia B. Atypical facial pain C. Wisdom tooth extraction D. Tonsillectomy E. Pharyngeal cancer pain
- Answer: C
Explanation:
(Raj Pain Review, 2nd Ed.,)
Primary (idiopathic) and secondary (oropharyngeal
cancer) glossopharyngeal neuralgia (cranial nerve 9) are
indications for the block of this nerve. Atypical facial pain
and tonsillectomy (pre-emptive analgesia) are also
indications. Maxillary nerve block is indicated for upper
teeth extraction and mandibular is indicated for lower
teeth extractions.
Source: Shah RV, Board Review 2003
924. Sphincter detrusor dyssynergia may respond to transsacral stimulation at: A. S1 nerve stimulation B. S2 nerve stimulation C. S3 nerve stimulation D. S4 nerve stimulation E. S5 nerve stimulation
- Answer: C
Source: Racz G. Board Review 2003
- A patient complains of worsening chronic headache,
despite treatment with aspirin, butalbital, caffeine and
ergotamine. MRI of the head was normal, but MRI of the
neck demonstrated spondylosis. Headache most likely is
due to:
A. Migraine
B. Drug rebound
C. Cervical spondylosis
D. Pseudo-tumor cerebri
E. Vasodilation due to ergotamine
- Answer: B
- A middle aged woman complains of abdominal pain that
began at age 8. Multiple medical and surgical evaluations
have been completely within normal limits. The patient
states that she has “always been sickly” and that her
mother “had the same problem.” There is no history of
childhood trauma or abuse. She has been unable to work
for the last 4 years. Which of the following psychological
diagnoses best describes her condition?
A. Somatization disorder
B. Conversion disorder
C. Hypochondriasis
D. Major depression
E. Generalized anxiety disorder
- Answer: A
Explanation:
A. The patient has a somatization disorder (also called
hysteria or Briquet’s syndrome).
* Patients often present with a long history of physical
complaints before 30 years of age.
- Many have undergone comprehensive medical
evaluations and surgical interventions without diagnosis
of any signifi cant disease process.
- They also have impaired social development because of their perceived illness.
- Occupational development is also affected.
- Many do not work or work at jobs limited by their
perceived pain.
B. In conversion disorder, there is loss of a physical
function that is temporarily related to a psychosocial
stressor.
* Sexual dysfunction, pain, blindness, and paralysis have
been described as manifestations of the psychological
confl ict experienced by patients with conversion disorder.
C. Hypochondriasis is the excessive preoccupation with
disease and with one’s health. Hypochondriacs believe that
a disease process exists despite medical evaluation and
reassurance over long periods of time.
* Patients must pay obsessive attention to perceived pain
symptoms without signifi cant fear or depression.
- They complain to family and physicians and are not
reassured by normal medical examinations and test results.
D. Major depression is symptomatically different from
somatization disorder.
E. Generalized anxiety disorder is symptomatically
different from somatization disorder.
- The daughter of a 65-year-old man describes her father as
having changed from an active, vivacious, caring person
to one who occasionally has trouble learning new facts,
has very little motivation to do any activity, and rarely
expresses feelings or emotions for his grandchildren
whom he has adored. The area of the brain most apt to be
involved in this type of behavior change is the
A. Hypothalamus
B. Reticular activating system
C. Heteromodal association areas
D. Limbic system
E. Unimodal association areas
- Answer: D
Explanation:
(Carlson, pp 91-94.)
· The limbic system includes regions of the limbic cortex,
as well as a group of interconnected structures that
surround the core of the forebrain.
· The limbic system forms a circuit whose primary
function was formerly regarded as modulating motivation
and emotional responses.
· Studies have discribed that the hippocampal formation
and the limbic cortex that surround it are involved in
learning and memory, rather than emotional behavior.
However, the remaining sections of the limbic system are
responsible for emotions, feelings, moods, and motivation.
Thus, limbic system is the site primarily responsible for
his learning diffi culty, lack of motivation, and emotional
feelings.
Source: Ebert 2004
- Which of the following is false with respect to tennis
elbow?
A. Forearm fl exors are typically involved
B. The involved muscles have tendinous attachments to the
lateral epicondyle of the humerus
C. The backhand stroke may be impaired
D. Corrective action includes loosening tight racquet
strings
E. Corrective action includes enlarging the racquet grip
- Answer: A
Explanation:
(Shah, Musculoskeletal Examination Presentation)
The wrist extensors are involved, typically due to
overuse/infl ammation/degeneration at their insertion on
the lateral epicondyle of the humerus. Forearm fl exors are
involved in medial epicondylitis, ‘golfer’s’ elbow. The
above corrective actions are true
Source: Shah RV, Board Review 2004
929. A celiac plexus block is not indicated for: A. Pancreatic cancer B. Chronic pancreatitits C. Sigmoid colon diverticulitis D. Hepatic metastases E. Chronic cholecystitis
- Answer: C
Explanation:
(Raj, Pain Review 2nd ed.)
The classic indication for celiac plexus block and
neurolysis is pancreatic cancer. The liver and gallbladder
are also indicated. The sigmoid colon is innervated by the
lumbar sympathetic chain Sympathetic innervation to the
gut distal to the mid-transverse colon is supplied by the
lumbar sympathetics. Note that the celiac plexus contains
both parasympathetic and sympathetic fi bers. However,
preganglionic sympathetics coalesce to form the greater
splanchnic (T5-T9) and lesser splanchnic (T10-T11)
nerves. These do not synapse in the sympathetic chain but
synapse in the celiac, aortico-renal, and superior
mesenteric ganglia.
Source: Shah RV, Board Review 2003
- The most common complication associated with a
supraclavicular brachial plexus block is
A. Blockade of the phrenic nerve
B. Intravascular injection into the vertebral artery
C. Spinal blockade
D. Blockade of the recurrent laryngeal nerve
E. Pneumothorax
- Answer: E
Explanation:
The most common complication associated with a
supraclavicular brachial plexus block is pneumothorax.
Other potential complications include phrenic nerve
paralysis, Horner’s syndrome, nerve damage or neuritis, or
intravascular injection.
- A 59-year-old female with mild CHF is treated with
furosemide. What is its primary mechanism of action?
A. Inhibition of sodium-potassium (Na+, K+) adenosine
triphosphatase (ATPase)
B. Inhibition of Na+, K+, choloride (Cl-) co-transporter
C. Inhibition of Na+, Cl- co-transporter
D. Inhibition of Cl- transporter
E. Inhibition of Ca2+ divalent cation (Ca2+) transporter
- Answer: B
Explanation:
The primary action of furosemide is inhibition of the Na+,
K+,Cl- transporter in the thick ascending limb of the loop
of Henle
Source: Hardman, p 697
- The most frequent work-related musculoskeletal disorder
found in the upper extremity is:
A. Carpal tunnel syndrome
B. Tendopathy of the extensor carpi radialis brevis
C. Posterior interosseus nerve entrapment
D. Shoulder external impingement
E. Cubital tunnel Syndrome
- Answer: A
Source: Sizer et al - Pain Practice - March & June 2004
- True statement concerning phantom limb pain is:
A. Trauma amputees have a higher incidence of phantom
limb pain than nontrauma amputees
B. The incidence of phantom limb pain increases with more
distal amputations
C. Nerve blocks are commonly used to treat phantom limb
pain
D. Most amputees do not experience phantom limb pain
E. Most phantom limb pain becomes more severe with
time
- Answer: C
Explanation:
The incidence of phantom limb pain is estimated to be 0
- 88%. The incidence of phantom limb
pain does not differ between traumatic and nontraumatic
amputees.
The incidence of phantom pain increases with more
proximal amputation.
Although very diffi cult to treat, and there is no clinical
evidence nerve blocks are commonly used in an attempt to
treat phantom pain. These include trigger point injections,
peripheral and central nerve blocks, and sympathetic
blocks.
- A 60-year-old man ambulates with his upper torso
stooped forward. His feet shuffl e and he has lost his arm
swing. Choose correct description of gait:
A. Ataxic gait
B. Parkinsonian gait
C. Spastic hemiplegic gait
D. Steppage gait
E. Scissor gait
- Answer: B
Explanation:
(Seidel, Sle, pp 791-792.)
A. Ataxic gait is often characterized by clumsiness; when
steps are taken, the advancing foot is lifted high.The foot is
then brought down in a slapping or stamping manner.
B. Parkinsonian gait is noted for the forward stoop of the
head and shoulders, with arms slightly abducted and
forearms partially fl exed; there is decreased arm swing as
the feet shuffl e.
C. Spastic hemiplegic gait is the result of spasticity of the
involved limb. The limb is moved forward by abduction
and circumduction.
D. Steppage gait occurs with footdrop (paralysis of the
peroneal nerve); the affected foot is raised higher than
normal to prevent dragging of the toe. Bilateral footdrop
results in a gait resembling that of a high-stepping horse.
E. Spastic diplegia gait or scissor gait occurs with
extrapyramidal disorders. The patient uses short steps and
drags the foot; the legs are extended and stiff and cross on
each other.
- Peripheral nerve stimulation for CRPS II:
A. peripheral nerve stimulation is more effective than spinal
cord stimulation
B. peripheral nerve stimulation and spinal cord stimulation
together is better than either alone
C. peripheral nerve stimulation should be used in mononeuropathy
D. psychological assessment should be done to rule out
contraindications
E. All of the above
- Answer: E
Source: Racz G. Board Review 2003
936. Most common indication for gasserian ganglion block is A. Glossopharyngeal neuralgia B. Atypical facial pain C. Trigeminal neuralgia D. Migraine headache E. Tension headache
- Answer: C
Source: Raj, Pain Review 2nd Edition
- Patrick’s test is a common physical exam technique to
elicit pain in which condition?
A. Sacro-iliac joint mechanical dysfunction and pain
B. Radicular pain due to lumbar spinal stenosis
C. Radicular pain due to a lumbar disc protrusion
D. Lumbar facet arthropathy
E. Lumbar discogenic pain
937. Answer: A Explanation: (Bonica, 3rd Ed., page 1587; Raj, Pain Review, 2nd Ed., page 139) Source: Shah RV: 2003
- A 22-year-old woman presents with the chief complaint
of diplopia for several weeks. She admits to occasional
vertigo and ataxia. Six months ago, she had urinary
incontinence for 1 month. Examination of the eyes
reveals nystagmus, and funduscopic exam reveals
swelling of the optic nerve (papillitis). The patient has
increased muscle tone of the lower extremities and is
hyperrefl exic. She has bilateral extensor plantar refl exes
and loss of position sense. Which of the following is the
most likely diagnosis?
A. Multiple sclerosis
B. Friedreich’s ataxia
C. Acute transverse myelitis
D. Brown-Sequard syndrome
E. Syringomyelia
- Answer: A
Explanation:
A. The patient most likely has multiple sclerosis, a
demyelinating disease characterized by visual impairment,
an afferent pupillary defect (Marcus Gunn pupil),diplopia,
nystagmus, limb weakness, spasticity, hyperrefl exia,
extensor plantar refl exes, vertigo, ataxia, dysarthria,
scanning speech, emotional lability, and bladder
dysfunction.
Patients with optic neuritis are at risk for developing
blindness.
B. Friedreich’s ataxia is an autosomal recessive disease in
which young patients present with pes cavus foot
deformity, spasticity, arefl exia, ataxia, and cardiomyopathy.
C. Patients with acute transverse myelitis initially present
with back pain followed by weakness and loss of sensation
below the level of the pain.
Often, there may be bladder and bowel incontinence. Transverse myelitis may be seen after vaccination or
infections.
D. Brown-Sequard syndrome (cord hemisection) is
characterized by contralateral loss of pain and temperature
and ipsilateral spasticity, weakness, hyperreflexia, extensor
plantar reflex, and loss of proprioception (vibration and
position sense).
E. Patients with syringomyelia have bilateral paralysis,
muscle atrophy, and fasciculations along with pain and
temperature sensory loss in a shawl-like or capelike
distribution.
Source: Tierney, 42/e, pp 983-984.
- A patient experienced a prolonged stay in one position
during a recent surgery and postoperative recovery that
resulted in compression of the common peroneal nerve
against the fi bular head. Which of the following motor
defi cits would be most likely to occur?
A. Loss of extension at the knee
B. Loss of plantar fl exion
C. Loss of fl exion at the knee
D. Loss of eversion
E. Loss of medial rotation of the tibia
- Answer: D
Explanation:
Compression of the common peroneal nerve would affect
all muscles innervated by this nerve, including tibialis
anterior, peroneus longus, and extensor digitorum longus.
Loss of dorsifl exion and eversion is usually complete. The
extensors of the knee joint (quadriceps femoris) are
supplied by the femoral nerve, whereas the fl exors of the
knee joint (the hamstrings and gracilis) are supplied by the
tibial nerve and obturator nerve, respectively. The
gastrocnemius and soleus muscles are the principal plantar
flexors of the foot and are innervated by the tibial nerve.
The popliteus is the prime medial rotator of the tibia and
is also innervated by the tibial nerve.
Source: Klein RM and McKenzie JC 2002.
- A patient presents to the emergency room 18 hours
after recovering from a spinal anesthetic, in which 5%
lidocaine was used. He complains of moderate to severe
pain in the lower back, buttocks, and posterior thighs.
The neurological and genito-urinary exams are normal. A
lumbar spine MR is normal. What is this patient suffering
with?
A. cauda equina syndrome
B. anterior spinal artery syndrome
C. transient radicular irritation
D. spontaneous intracranial hypotension
E. epidural hematoma
- Answer: C
Explanation:
(Raj, Practical Mgmt of Pain, 3rd Ed., page 371; Stoelting,
Pharmacology and Physiology of Anesthetic Practice, 3rd
Ed., page 168-169)
A. Given the absence of any neurological or GU signs, one
would most likely suspect transient radicular irritation
and exclude cauda equina syndrome.
B. Anterior spinal artery syndrome often presents with
isolated leg weakness.
C. Hyperbaric lidocaine that is injected intrathecally can
present as severe low back, buttock, and groin pain,
secondary to transient neurological irritation. Treatment
is conservative.
D. Spontaneous intracranial hypotension presents with
headache and neurological symptoms.
E. The MRI would exclude a new epidural hematoma.
Source: Shah RV, Board Review 2004
941. Increased activity of the sympathetic nervous system causes A. Penile erection B. Pupillary constriction C. Accommodation for near vision D. Bronchiolar dilation E. Gallbladder emptying
- Answer: D
Explanation:
(Rhoades, pp 118-120.)
Activation of the sympathetic nervous system produces relaxation of the smooth muscles surrounding the
bronchioles, leading to bronchiolar dilation. The
parasympathetic nerves are responsible for penile erection,
pupillary constriction, contraction of the ciliary muscle
during accommodation for near vision, and
gallbladder emptying. Sympathetic stimulation causes
ejaculation and pupillary dilation but does not affect the
activity of the ciliary muscle or the gallbladder.
- A 51-year-old alcoholic presents to the emergency room
with horizontal nystagmus, ataxic gait, and confusion.
Which of the following is the most likely diagnosis?
A. Wernicke syndrome
B. Niacin deficiency
C. Korsakoff syndrome
D. Kliiver-Bucy syndrome
E. Delirium tremens
- Answer: A
Explanation:
(Tierney, 42/e, p 985.)
The triad of nystagmus and paralysis of eye muscles,ataxia,
and confusion is associated with Wernicke syndrome.
Korsakoff syndrome consists of confabulation, confusion,
and recent memory loss.These disorders are often found in
thiamine (B1) defi cient malnourished alcoholics and are
secondary to lesions in the mamillary bodies. Niacin
defi ciency (pellagra or vitamin B3 defi ciency) causes the
triad of D’s (Dementia, Dermatitis, and Diarrhea). Kluver-
Bucy syndrome is due to lesions in the amygdala; patients
present with hypersexuality, compulsive attention to detail,
docile behavior, and an inability to recognize objects
visually (agnosia). Delirium tremens is seen 48 to 96 h
following abstinence from alcohol; patients present with
insomnia, confusion, tremors, delusions, visual
hallucinations, and hyperactivity of the autonomic
nervous system (i.e., sweating, tachycardia, fever,and
dilated pupils).
- The condition in which the covering of the spinal
cord, along with enclosed neural tissue, forms a saclike
projection through a dorsal defect in the vertebral
column is termed
A. Rachischisis
B. Anencephaly
C. Meningocele
D. Meningomyelocele
E. Hydrocephaly
- Answer: D
Explanation:
In the family of conditions known as spina bifi da,failure of
the dural portions of the developing vertebrae may expose
a portion of the spinal cord and its covering. This usually
occurs near the caudal end of the neural tube.If there is no
projection of the spinal cord or its covering through the
bony defect, the condition is generally hidden (spina bifi da
occulta). However, it is termed spina bifi da cystica when
spinal material traverses the defect.
A. Rachischisis is an extreme example of spina bifi da
cystica in which the neural folds underlying the vertebral
defect fail to fuse, leaving an exposed neural plate.
B.Anencephaly occurs when the cranial neural tube fails to
fuse, thus resulting in lack of formation of forebrain
structures and a portion of the enclosing cranium.
C. In a meningocele, this is a saclike projection formed
only by the meninges.
D. If the projection contains neural material, it is a
meningomyelocele.
E. Hydrocephaly results from blockage of the narrow
passageways between the ventricles or between the
ventricles and the subarachnoid space. Resultant swelling
of the ventricles compresses the brain against the cranial
vault and may cause serious mental defi cits.
944. The most appropriate drug for reversing myasthenic crisis in a patient who is experiencing diplopia, dysarthria, and difficulty swallowing is A. Neostigmine B. Pilocarpine C. Pralidoxime D. Succinylcholine E. Tubocurarine
- Answer: A
945. A supraclavicular brachial plexus block, blocks the following section of the plexus: A. Roots B. Trunks C. Divisions D. Cords E. Branches
- Answer: B
Explanation:
The advantages of the supraclavicular block are fourfold.
The plexus is blocked where it is most compact, namely at
the level of the trunks.
A small volume of anesthetic is required and no part of the
plexus is spared as with axillary or interscalene block.
The block can be performed with the arm in any position.
- A patient presents with right low back and hip pain
following a motor vehicle accident several weeks ago.
Pain is made substantially worse with internal rotation of
the right lower extremity. Hip fl exion and extension are
not painful. MRI demonstrates an L4/5 disc herniation.
The source of pain is most likely arises from which of the
following structures?
A. Disc
B. Facet joint
C. Hip joint
D. Sacroiliac joint
E. Piriformis muscle
- Answer: E
- A college student has a headache history of 3 months.
Headache is bilateral, constricting with nausea, but no
vomiting. Physical and neurological exams are normal.
The drug of choice is:
A. Acetaminophen
B. Oxycodone
C. Gabapentin
D. Amitriptyline
E. Sumatriptan
- Answer: A
Explanation:
The patient most likely has tension headache.
948. A patient with rectal cancer with infi ltration develops a new onset of low back pain. He is on oxycodone and antidepressant therapy. For treatment of low back pain, the recommended addition is as follows: A. Ibuprofen B. Gabapentin C. Mexiletine D. Morphine E. Transdermal Fentanyl
- Answer: A
- A 25-year-old woman is involved in a motor vehicle
accident. Among her injuries is a lumbar vertebral body
fracture. Which of the following most likely contributed
to this injury?
A. Flexion
B. Extension
C. Torsion
D. Spondylolisthesis
E. Subluxation
- Answer: A
Explanation:
the person is upright. Fracture of a lumbar vertebral body
may be seen in vehicular accidents when the victim is
restrained during a high-speed impact by a seat belt
without a shoulder harness. The rapid and extreme
forward fl exion of the lumbar spine may produce a variety
of spinal injuries, ranging from fractures to dislocations.
Fractures suffered during falls in which the person is
upright, such as may occur when someone jumps off a
building, are usually compression fractures of the
vertebral body. Fracture of the vertebral body will usually
produce pain coincidental with the injury. Patients with
fractures of the vertebral body that occur without trauma
or with inconsequential trauma must be investigated for
malignant processes, such as metastatic carcinoma,
multiple myeloma, and unsuspected osteomyelitis.
Source: Anschel 2004
- The carpal bone that is most likely to dislocate anteriorly
and cause a form of carpal tunnel syndrome is the
A. Capitate
B. Hamate
C. Lunate
D. Navicular
E. Scaphoid
- Answer: C
Explanation:
A. The capitate is frequently fractured but does not tend to
dislocate into the carpal arch.
B. The hamate provides an anchor for the transverse carpal
ligament and is, therefore, located lateral to the carpal
tunnel.
C. The lunate bone tends to dislocate anteriorly into the
transverse carpal arch, thereby entrapping the tendons of
the extrinsic digital fl exors and compressing the median
nerve.
D, E. The navicular (scaphoid) bone has a tendency to
fracture but does not dislocate into the carpal tunnel.
- A 35-year-old man has acute onset of low back pain, lower
extremity weakness, and bladder dysfunction. He had a
lumbar laminectomy two years ago. A myelogram shows
disc herniation L4-5. The most appropriate management
is
A. Bed rest
B. Administration of nonsteroidal anti-inflammatory agent
C. Epidural administration of a corticosteroid
D. Epidural administration of a local anesthetic
E. Surgical decompression
- Answer: E
952. Clinical fi ndings due to S-1 radiculopathy include: A. Absent ankle (Achilles) refl ex B. Weakness of foot dorsifl exion C. Neurogenic bladder D. Positive unilateral Babinski sign E. All of the above
- Answer: A
Explanation:
With S-1 radiculopathy, there is reduction of ankle refl ex
due to gastrocnemius muscle weakness. Dorsifl exion of
foot is normal, as this involves the L-4 and L-5 roots.
Neurogenic bladder is seen with spinal cord or S-2, S-3,
and S-4 root involvement. Babinski sign is seen with
spinal cord, not spinal root lesions.
Source: Neurology for the Psychiatry specialty Board
Review By Leon A. Weisberg, MD
953. A herniated T-8 thoracic disk may cause which of these findings: A. Paraparesis B. Autonomic bladder C. Bilateral Babinski signs D. Absent abdominal refl exes E. All of the above
- Answer: E
Explanation:
A herniated T-8 thoracic disk may compress the thoracic
spinal cord, causing all the listed neurological
disturbances. It can also cause thoracic radiculopathy
resulting in bandlike sensory disturbance in the thoracic
or abdominal region. This latter pattern may simulate
shingles (herpes zoster without rash).
Source: Neurology for the Psychiatry specialty Board
Review By Leon A. Weisberg, MD
- In Hirschsprung’s disease, neural crest cells fail to migrate to, or invade, the wall of the lower colon, resulting in a loss of peristalsis in that region and often fatal obstruction. Preganglionic neurons, which would innervate the absent intramural ganglia, originate in
A. The nucleus ambiguus
B. Cervical intermediolateral cell column
C. Sacral levels two to four of the spinal cord
D. The motor nucleus of the vagus nerve
E. The ventral horn at spinal levels T12, L1, L2
- Answer: C
Explanation:
A. The nucleus ambiguus is the source of preganglionic
parasympathetic neurons that innervate the heart via the
vagus nerve and cardiac plexus.
B. Neurons arising in the cervical intermediolateral cell
column are sympathetic preganglionics.
C. Preganglionic parasympathetic neurons to the lower
colon arise from the spinal cord at sacral levels two to four
and reach the wall of the colon via pelvic splanchnic
nerves.
D. Preganglionic parasympathetic neurons arising from
the motor nucleus of the vagus innervate the upper GI
tract.
E. Neurons arising from the ventral horn are primary
somatic motor neurons to skeletal muscle.
- A 32-year-old man was admitted for neurologic
evaluation of a gun-shot wound received fi ve days
previously A 9-mm bullet had passed through both the
medial and lateral heads of the gastrocnemius muscle.
The bullet had not struck bone or signifi cant arteries.
Neurologic examination revealed losses of dorsifl exion
and eversion of the left foot. The patient could not feel
pinprick or touch on the dorsum of the left foot or
anterolateral surface of the left leg. Which nerve was most
likely involved in the injury?
A. Sciatic nerve
B. Femoral nerve
C. Sural nerve
D. Common peroneal nerve
E. Tibial nerve
- Answer: D
Explanation:
A. The sciatic nerve generally divides into the tibial and
common peroneal nerves superior to the popliteal fossa.
Damage to it might result in defi cits in both plantar
fl exion and dorsifl exion.
B. The femoral nerve innervates the quadriceps muscles of
the anterior thigh.
Damage to it would impair fl exion of the thigh at the
hip.
C. The common peroneal nerve innervates all muscles in
the anterior and lateral compartments of the leg.
The common peroneal nerve provides sensory
innervation to the dorsum of the foot and the
anterolateral surface of the legs via the superfi cial and
sural/lateral sural cutaneous nerves, respectively.
D. The common peroneal nerve is the lateral terminal branch of the sciatic nerve.
After arising near the apex of the popliteal fossa, it
descends on the popliteus muscle and winds superficially
around the fibular neck.
It is extremely vulnerable in this position and is the
most often injured nerve in the lower extremity.
E. The tibial nerve innervates plantar flexors of the
posterior compartment.
956. A tumor in the infratemporal fossa may gain entrance to the orbit through which of the following? A. The optic foramen B. The ethmoidal sinuses C. The pterygoid canal D. The inferior orbital fi ssure E. The superior orbital fi ssure
- Answer: D
Explanation:
A, E. The optic foramen and superior orbital fi ssure open
into the middle cranial fossa and transmit the optic nerve
and the oculomotor, trochlear, and abducens nerves,
respectively.
B. The ethmoidal sinuses are mucosa-lined cavities within
the ethmoid and adjacent bones. They drain into the nasal
cavity.
C. The pterygoid canal connects the middle cranial fossa
with the pterygopalatine fossa and transmits the vidian
nerve.
D. The infratemporal fossa communicates directly with
the orbit via the inferior orbital fi ssure and the
pterygopalatine fossa.
The fi ssure normally transmits branches of the
maxillary nerve and branches of the infraorbital
vessels.
- Which statement regarding cervical nerve roots is true:
A. The C7 spinal nerve exits through the C7-T1 foramen
B. The C2 spinal nerve exits through the C1-2 neuroforamen.
C. Sensory innervation to the occiput is supplied primarily
by branches from C1
D. The greater occipital nerve originates from the ventral
root of C2
E. The C6 and C7 spinal nerves are most commonly involved in cervical radiculopathy
- Answer: E
Explanation:
References:
2.Gray’s Anatomy, Thirteenth American Edition. Page
960.
3.The Anatomic Relation Among the Nerve Roots,
Intervertebral Foramina, and Intervertebral Discs of the
Cervical Spine
4.Tanaka, The Anatomic Relation Among the Nerve Roots,
Intervertebral Foramina, and Intervertebral Discs of the
Cervical Spine SPINE 2000;25:286-291
5.Mercer, The Ligaments and Anulus Fibrosus of Human
Adult Cervical Intervertebral Discs, SPINE 1999;24:619
There are 7 cervical spinal levels and 8 cervical spinal
nerves. The fi rst two cervical nerves (C1 and C2) exit the
spinal canal posterior to the atlanto-occipital and atlantoaxial
joints respectively. These two nerves do not exit via a
foramen. The fi rst cervical foramen is C2-3 which
transmits the C3 nerve. From C2-3 to C7-T1, the spinal
nerve exiting the foramen is named by the last number of
the level (i.e. C3 exits the C2-3 foramen, C4 exits the C3-4
foramen and C7 exits the C6-7 foramen). The C8 nerve
exits the C7-T1 foramen. Below T1, the numbering
convention reverses and the exiting nerve is named for the
fi rst number of the level (i.e. T2 exits the T2-3 foramen
and L4 exits the L4-5 foramen).
The greater occipital nerve is the medial branch of the
dorsal primary ramus of C2. It supplies most of the
sensory innervation to the occiput. The C1 spinal nerve is
primarily motor.
Degenerative changes of the intervertebral discs and nerve
root impingement in the intervertebral foramen occur
most commonly at the C5–C6 and C6–C7 levels. Kelsey et
al investigated the epidemiology of prolapsed cervical
discs in an attempt to provide descriptive statistics on this
disorder and to identify possible risk factors. Most
patients (75%) had prolapsed discs at either the C5–C6 or
C6–C7 level. Likewise, according to Murphey et al, the
frequency of cervical radiculopathy was 26% for C6, 61%
for C7, and 8% for C8. The incidence of nerve root
compression is high for C6 and C7.
Cadaveric dissection data from Tanaka et. Al. predicts a
higher incidence of radiculopathy for the C5, C6 and C7
nerve roots. The C5 nerve roots were found to exit over
the middle aspect of the intervertebral disc,whereas the C6
and C7 nerve roots were found to traverse the proximal
part of the disc. The C8 nerve roots had little overlap with
the C7–T1 disc in the intervertebral foramen. The C6 and
C7 rootlets passed two disc levels in the dural sac. Also, a
high incidence of the intradural connections between the
dorsal rootlets of C5, C6, and C7 segments was found.
Source: Schultz D, Board Review 2004
- Pattern of limitation most frequently accompanying
subacromial impingement
A. Glenohumeral abduction limits
B. Glenohumeral adduction limits
C. Glenohumeral external rotation limits
D. Glenohumeral internal rotation limits
E. Glenohumeral external and abduction limits
- Answer: D
Source: Sizer Et Al - Pain Practice March & June 2003
- Intervertebral disks have a tendency to herniate into the
intervertebral foramen because the
A. Annulus fi brosus is attenuated in the posterolateral
regions
B. Interspinous ligament reinforces the disks anteriorly and
anterolaterally
C. Ligamentum fl avum reinforces the intervertebral disks
posteriorly
D. Lumbar intervertebral disks are thicker posteriorly than
anteriorly
E. Posterior longitudinal ligament is stronger and more
complete posteriorly than posterolaterally
- Answer: E
Explanation:
(Moore, Anatomy, 4/e, pp 451-453.)
Intervertebral disks are strongly reinforced ventrally
and laterally by the anterior longitudinal ligaments. The
posterior longitudinal ligament, although it is denticulate
and attenuated laterally, reinforces the posterior aspect of
the intervertebral disk. Because the posterolateral region of the disk is supported least by ligamentous structures, a
nucleus pulposus that is herniated through the annulus
fibrosus of the intervertebral disk will take the line of least
resistance and move posterolaterally into the
intervertebral foramen.In so doing, the herniation is apt to
impinge on a spinal nerve of the next lower vertebral
level.
Source: Klein RM and McKenzie JC 2002.
- A 75-year-old female in congestive heart failure (CHF)
is unable to climb a fl ight of stairs without experiencing shortness of breath. Digoxin is administered to improve
cardiac muscle contractility. Within two weeks, she has
marked improvement in her symptoms. What cellular
action of digoxin accounts for this?
A. Inhibition of cyclic adenosine 5’-monophosphate
(cAMP) synthesis
B. Inhibition of mitochondrial calcium (Ca2+) release
C. Inhibition of the sodium (Na+) pump
D. Inhibition of b-adrenergic stimulation
E. Inhibition of adenosine triphosphate (ATP) degradation
- Answer: C
- The DSM-IV classifi cation of psychiatric disorders
represent a major advance in psychopathology by
A. Detailing the treatments for various mental disorders
B. Predicting the outcome of less severe psychological
problems
C. Evaluating the effi cacy of various drug treatments
D. Assessing the potential etiology of abnormal behaviors
E. Defi ning by empirical criteria a wide variety of psychiatric
disorders
- Answer: E
Explanation:
(Baum, pp 264-271.)
The DSM-IV is a multiaxial classifi cation and
categorization manual for a wide variety of psychological
disorders. One of its newer contributions is that each
individual is scored according to broad categories and axes
so that an individual may be classifi ed as having several
disorders rather than being forced into a single category or disorder.
The various categories of axes are Axis I: Primary psychiatric disorders (including physical
and sexual abuse, medication-induced disorders,
noncompliance)
Axis II: Personality disorders and mental retardation (can
also be used for maladaptive personality and defense
mechanisms)
Axis III: General medical conditions (general physical
health important to the total diagnostic picture)
Axis IV: Psychosocial and environmental problems
(family, personal, or situational problems that might affect
the diagnosis, treatment, or program)
Axis V: Global assessment of functioning (a scale of the
level of functioning at the time of evaluation and at other
time periods)
Source: Ebert 2004
962. Which of the following conditions mimics thalamic pain syndrome? A. Wallenberg’s syndrome B. Syringomyelia C. Lateral medullary syndrome D. Parietal cortical lesion E. Lumbar Radioculopathy
- Answer: B
Source: Raj P, Pain medicine - A comprehensive Review -
Second Edition
963. Acute Herpes zoster infection (shingles) of 3 week’s duration is most appropriately treated by which of the following? A. Topical lidocaine patch B. Peripheral nerve blockade C. Topical capsaicin cream D. Spinal cord stimulation E. Intrathecal steroids
- Answer: B
Explanation:
Lidocaine patches should only be used on intact skin.
Similarly, capsaicin should not be used on broken skin.
Peripheral nerve blockade, such as intercostal blocks, is
the best choice of the options provided. SCS and
intrathecal steroids may be appropriate for postherpetic
neuralgia.
- Among combat veterans, the greatest risk for
posttraumatic stress disorder is among those who
A. Were violent prior to service
B. Have a prior history of depression
C. Have coexisting sociopathy
D. Participated in violence towards noncombatants
E. Have a history of substance abuse
- Answer: D
Explanation:
(Sierles, pp 264-266. Ebert, pp 341-350.)
Posttraumatic stress disorder (PTSD) is a cluster of
symptoms that can occur in a person after exposure to a
severely stressful event (e.g., rape, combat, natural
disaster). There are three categories of symptoms:
reexperiencing the event (e.g., nightmares, daydreams,
obsessions, fl ashbacks), withdrawal (e.g., avoiding movies
about war and rape and feeling detached from others who
have not experienced the event), and hyperarousal (e.g.,
insomnia, irritability, hypervigilance, severe anxiety). Most
patients with PTSD recover, especially those with good
premorbid functioning and support. The greatest risk for
PTSD among combat veterans is among those who killed
noncombatants, participated in atrocities, or were
wounded. Another factor associated with increased risk for
PTSD is violence or behavioral problems, sociopathy, or
psychiatric disorders prior to the trauma. Substance
abuse, including alcoholism, is also relevant. It is estimated
thats many as 480,000 American veterans of the war in
Vietnam have PTSD.
Source: Ebert 2004
- In general, modalities such as heat, electrical stimulation
and ultrasound:
A. Should be used until the patient is cured of their pain.
B. Are the best method to treat patients with chronic pain.
C. When combined with injections are the only thing necessary
to treat the majority of pain conditions.
D. Should be used to facilitate an active exercise program
for a short course.
E. Should never be used following interventional techniques
- Answer: D
Source: Malanga G, Board Review 2003
966. A 36-year-old man presents with left hand weakness and atrophy of the first dorsal interosseous muscle. This may indicate damage to spinal roots A. C5 and C6 B. C6 and C7 C. C7 and C8 D. C8 and T1 E. T1 and T2
- Answer: D
Explanation:
The fi rst dorsal interosseous muscle is innervated by the
ulnar nerve. The fi bers of the ulnar nerve reaching this
muscle originate a the C8 and T1 roots. If the ulnar nerve
itself is the neural element injured, it is usually because of
damage at the elbow, where the ulnar nerve runs
superfi cially in the groove over the ulnar condyle. All the
interosseous muscles of the hand are supplied by the ulnar
nerve: complete transection of that nerve will produce
interosseous wasting and impaired fi nger adduction and
abduction. Although the lumbrical muscles are situated
alongside the interosseous muscles of the hand, only two
lumbricals – those on the ulnar metacarpals – are
innervated by the ulnar nerve. The other two lumbricals
are innervated by the median nerve. All four lumbricals
insert on the extensor sheaths of the fi ngers and
participate in extension of the digits.
Source: Anschel 2004
- A 49-year-old woman is brought to the emergency room
after suddenly losing consciousness. Her husband states
that the patient was in good health until 2 h ago, when
she suddenly complained of a severe headache. After
one episode of vomiting, the patient lost consciousness.
The husband states that there were no seizure-like
movements and no incontinence. The patient did not
take any medications, smoke, drink, or use illicit drugs.
On physical examination, the patient has a regular heart
rate of l00/min, respiratory rate of 16/min, and blood
pressure of 120/80 mmHg, and is afebrile. Heart and
lung examinations are normal. On neurologic exam, the
patient responds only to painful stimuli and her deep
tendon refl exes are bilaterally equal. She has bilateral
fl exor plantar responses. She has neck stiffness and
attempts to resist forward fl exion. Which of the following
is the most likely diagnosis?
A. Carotid artery thrombosis
B. Embolic infarction of the brain
C. Frontal lobe hemorrhage
D. Subarachnoid hemorrhage
E. Complicated migraine
- Answer: D
Explanation:
(Tierney, 42/e, pp 961-967.)
There are three types of stroke: subarachnoid hemorrhage,
cerebral infarction, and intracerebral hemorrhage. This
patient presented after complaining of a severe headache.
She has neck stiffness and no focal defi cit on neurologic
exam. The loss of consciousness requires bihemispheral
dysfunction, and this along with the abrupt history is
most consistent with a subarachnoid hemorrhage (SAH).
Common causes of SAH include ruptured aneurysm (i.e.,
berry) and arteriovenous malformation (AVM).
Intracerebral hemorrhage (ICH) rarely produces coma
(must be signifi cantly large to do so), and patients do not
complain of headache (does not involve the meninges).
Patients with ICH have focal defi cits that appear abruptly
slowly progress over hours. An embolic stroke can involve
any carotid artery but must be bilateral to cause loss of
consciousness. Patients have a history of atrial fi brillation
or cardiac problems.
- During a C7 stellate ganglion block, 2 cc of bupivacaine
with epinephrine were injected. The patient developed
myoclonic activity and lost consciousness. The injection
most likely was into the
A. Subdural space
B. Vertebral artery
C. Epidural vein
D. Subarachnoid space
E. Internal jugular vein
- Answer: B
- A 62-year-old man walks with his feet widely spaced;
steps occur with each foot lifted abruptly and too high
and brought down in a stamping manner. Choose correct
description of gait:
A. Ataxic gait
B. Parkinsonian gait
C. Spastic hemiplegic gait
D. Steppage gait
E. Scissor gait
- Answer: A
Explanation:
(Seidel, Sle, pp 791-792.)
A. Ataxic gait is often characterized by clumsiness; when
steps are taken, the advancing foot is lifted high.
The foot is then brought down in a slapping or stamping
manner. Spastic hemiplegic gait is the result of spasticity
of the involved limb. The limb is moved forward by
abduction and circumduction.
B. Parkinsonian gait is noted for the forward stoop of the head and shoulders, with arms slightly abducted and
forearms partially fl exed; there is decreased arm swing as
the feet shuffl e.
Steppage gait occurs with footdrop (paralysis of the
peroneal nerve); the affected foot is raised higher than
normal to prevent dragging of the toe.
Bilateral footdrop results in a gait resembling that of a
high-stepping horse.
C. Spastic diplegia gait or scissor gait occurs with
extrapyramidal disorders.
The patient uses short steps and drags the foot; the legs
are extended and stiff and cross on each other.
- A 35-year-old woman with Complex Regional Pain
Syndrome I of the right upper extremity, develops miosis,
ptosis and enophthalmos after undergoing a stellate
ganglion block. She does not notice a signifi cant pain
relief. No signifi cant rise in skin temperature changes was
recorded to the right upper extremity. What is the most
likely cause?
A. Inadequate concentration of the local anesthetic
B. Intravascular injection
C. Subarachnoid block
D. Anomalous Kuntz nerves
E. Brachial plexus block
- Answer: D
Explanation:
The sympathetic supply to the upper extremity is
through the grey rami communicantes of C7, C8 and T1
with occasional contributions from C5 and C6.
This innervation is through the stellate ganglion. Blocking
the Stellate ganglion would effectively cause a sympathetic
denervation of the upper extremity.
In some cases the upper extremity maybe supplied by the
T2 and T3 grey rami communicantes. These fi bers do not
pass through the stellate ganglion.
These are Kuntz’s fi bers and have been implicated in
inadequate relief of sympathetically maintained pain
despite a good stellate ganglion block.
These fi bers can be blocked by a posterior approach.
Successful block of the sympathetic fi bers to the head is
indicated by the appearance of Horner’s syndrome.
Successful block of the sympathetic block of the upper
extremity is indicated by a rise in skin temperature,
engorgement of veins on the back of the hand, loss of skin
conductance response and a negative sweat test.
Source: Chopra P. 2004
971. A 35-year-old woman falls 12 ft off a ladder and fractures her c-spine, causing damage at the C4 level. She is initially a flaccid quadriplegic with arefl exia. This arefl exia and flaccidity usually evolve into hyperrefl exia and spasticity within A. 2 to 4 months B. 1 to 2 months C. 3 days to 3 weeks D. 1 to 3 h E. 5 to 25 min
- Answer: C
Explanation:
Spinal shock is a transient phenomenon that occurs with
damage to fi bers from upper motor neurons.The spasticity
that usually develops within a few days of the spinal cord
injury is presumed to represent exaggeration of the normal
stretch refl exes in the limbs disconnected from upper
motor neuron control. The evolution from spinal shock to
spasticity is much more typical of spinal cord injuries than
it is of cerebrocortical injuries, but even with
cerebrocortical injuries there is usually an interval of hors
to days during which limbs that eventually become
hyperrefl exic and spastic are hyprorefl exic and fl accid.
Source: Anschel 2004
- A 29-year old female with upper extremity complex
regional pain syndrome undergoes a stellate ganglion
block in your offi ce pain clinic. She is otherwise healthy
with normal body habitus and normal airway. She has
been NPO for 12 hours. 20cc of 0.25% bupivacaine is
injected incrementally over one minute with no other
medication administered. 5 minutes after injection
the patient complains of generalized weakness which
progresses to complete unresponsiveness, apnea and
hypotension over the ensuing several minutes.The
following is the most likely diagnosis:
A. Overdose of bupivacaine
B. Total spinal anesthesia
C. Spinal cord infarction
D. Anaphylactic shock
E. Vertebral artery injection
- Answer: B
Explanation:
Reference:
Gilbert, Complications and Controversies in Regional
Anesthesia, in ASA Refresher Course, Chapter 6, Volume
3, ASA 2003
Neural Blockade, Cousins and Bridenbaugh, Second
Edition, Chapter 22 Complications of Local Anesthetic
Neural Blockade, pp. 695-718.
Total spinal anesthesia refers to the condition in which an
overdose of intrathecal local anesthetic is administered,
resulting in blockade of the entire intraspinal neuraxis.
Patients with total spinal will manifest a complete and
total, albeit temporary, paralysis. Manifestations include:
Blockade of C3, C4 and C5 nerve roots (C3, 4 and 5
keep the diaphragm alive) as well as all thoracic spinal
nerves resulting in diaphragm and chest wall paralysis
with apnea.
Blockade of sympathetic fi bers with hypotension
secondary to vasodilation and bradycardia.
Complete muscle paralysis with loss of all voluntary
movement including speech an eye opening.
Unless hypotension is severe, the patient may remain
awake and aware but completely unable to respond. Total
spinal is the most likely diagnosis here because of the
signifi cant risk of dural root sleeve injection with stellate
ganglion block and the delayed and gradual onset of the
event, taking several minutes to develop.
The other choices can be eliminated as follows:
Overdose of bupivacaine:
20 cc of 0.25% bupivacaine contains 50 mg of
bupivacaine (one can easily calculate the mg/ml from the
milliliters and percent of any local anesthetic. Simply
multiply the percent (0.25) by 10. This will give the
number of mg per milliliter (2.5). Multiply this number
by the volume of 20ml to arrive at 50 mg).
The following are recommended maximum single doses
for common local anesthetics:
lidocaine: 300 mg without epinephrine, 500 mg with
epinephrine
bupivacaine: 175 mg to 225 mg
Although there are case reports of cardiac toxicity with
direct intravascular injection of as little as 50 mg of
bupivacaine, direct intravascular injection would have
resulted in immediate, not delayed effects. Soft tissue
infi ltration overdose of bupivacaine would require a dose
in the range of 175 mg.
Spinal cord infarction would be exceedingly unlikely
from an injection of plain local anesthetic and the time
course would be quicker.
Vertebral artery injection would cause immediate
seizures.
Anaphylactic shock is a possibility but unlikely with the
use of an amide local anesthetic.
Source: Schultz D, Board Review 2004
- A 36-year old felt a sharp pain in the neck, radiating to the dorsal aspect of the forearm when he was lifting a large
box. He started experiencing numbness of the thumb
and index fi nger, with decreased ability to perform biceps
fl exion. On examination, a diminished biceps refl ex was
found. What is the most likely cause of the patient’s
problems?
A. Fractured C5 vertebra
B. C5/6 disc protrusion
C. Facet syndrome at C5/6
D. Compression of the C5 nerve root by an osteophyte
E. C4/5 disc protrusion
- Answer: B
Explanation:
The patient has evidence of C6 root compression, most
likely due to C5/6 disc protrusion. Pain in the neck,
shoulder, medial scapula, anterior chest, lateral aspect of
the upper arm, and dorsal aspect of the forearm associated
with biceps and extensor carpi radialis weakness is
frequently present.The patient may complain of numbness
of the thumb and index fi nger. The biceps refl ex may be
diminished or absent (Wall, p 715)
Source: Kahn CH, DeSio JM. PreTest Self Assessment and
Review. Pain Management. New York, McGraw-Hill, Inc.,
1996.
- A type I diabetic patient has been treated with relaxation
techniques daily for one month. This treatment is likely
to affect the management of her diabetes by
A. Increased levels of plasma cortisol
B. Increased sensitivity to insulin
C. Increased glucose-stimulated secretion of insulin
D. Signifi cant improvement in glucose tolerance
E. No signifi cant change in requirements for exogenous
insulin
- Answer: D
Explanation:
(Taylor, pp 530-531.)
· The use of relaxation techniques to reduce stress has
proven very effective.
· Studies of diabetic patients who practiced progressive
muscle relaxation showed signifi cant improvement in
glucose tolerance following relaxation training.
· Plasma cortisol levels were also reduced in patients
trained in relaxation.
· Relaxation, however, did not affect insulin sensitivity or
glucose-stimulated secretion of insulin.
· Stress reduction techniques, such as relaxation, are
effective in reducing requirements for exogenous insulin
and in the management of both insulin-dependent and
non-insulin-dependent diabetes.
Source: Ebert 2004
- A 31-year-old female has been treated with fl uoxetine for
two months with no improvement in her depression. You
decide to switch antidepressant therapy to phenelzine and
instruct her to wait one week after stopping fl uoxetine to
start taking the new pills. She begins therapy immediately
with phenyline without discontinuing fl uoxetine. Two
days later, she is brought to the ED with unstable vital signs, muscle rigidity, myoclonus, and hyperthermia.
What caused these fi ndings?
A. Increased serotonin (5-HT) in synapses
B. Increased norepinephrine in synapses
C. Increased acetylcholine in synapses
D. Increased dopamine in synapses
E. decreased norepinephrine in synapses
- Answer: A
Explanation:
Reference: Hardman, p 444.
This patient has the serotonin syndrome.
Serotonin is already present in increased amounts in
synapses because of blockade of its reuptake by the SSRIs.
The amount of serotonin that is present further increased
when breakdown by MAO is inhibited.
The serotonin syndrome can be life threatening.
Source: Stern - 2004
- A 40-year old construction worker presents with pain
over the dorsal aspect of the forearm and inability to
fully extend the arm at the elbow. Physical examination
reveals diminished sensation over the dorsal aspect of
the index and middle fi ngers as well as an absent triceps
refl ex. The most likely diagnosis:
A. C5
B. C6
C. C7
D. C8
E. T1
- Answer: C
Explanation:
Pain in the posterior aspect of the arm is likely due to a C7
root lesions, whereas medial anterior or lateral arm pain
may be due to C6 or C7 nerve root lesions. A C7 nerve
root lesion will also produce symptoms (pain and
paresthesias) in the index and middle fi ngers as well as a
diminished or absent triceps refl ex. Absence of a
brachioradialis refl ex is an indication of a C6 nerve root
lesion (Raj, pp 272-273).
Source: Kahn CH, DeSio JM. PreTest Self Assessment and
Review. Pain Management. New York, McGraw-Hill, Inc.,
1996.
977. If nystagmus is a prominent symptom of a cerebellar lesion, the lesion is within A. The dentate nucleus B. The fl occulonodular lobe C. The lateral cerebellum D. The cerebrocerebellar cortex E. The superior cerebellar peduncle
- Answer: B
Explanation:
(Guyton, p 655.)
The fl occulonodular lobe is known as the
archeocerebellum because it is, phylogenetically, the oldest
portion of the cerebellum. It is connected to the vestibular
nuclei and participates in the control of eye movements.
Lesions to the fl occulonodular lobe will cause nystagmus.
Lesions to the other regions of the cortex, the deep nuclei
of the spinocerebellar tracts, cause a variety of
abnormalities in motor coordination referred to as ataxia.
- A patient presents with onset of upper extremity pain.
The physical examination revealed weakness of elbow
extension and loss of sensation of the middle fi nger. The
correct diagnosis in this patient is:
A. C4 nerve root involvement
B. C5 nerve root involvement
C. C6 nerve root involvement
D. C7 nerve root involvement
E. C8 nerve root involvement
- Answer: D
Explanation:
The C7 (radial nerve) supplies the triceps, which is the
primary elbow extensor while the triceps, wrist fl exors,
and fi nger extensors are partially innervated by the C8,
they are predominantly C7 muscles.
C7 supplies sensation to the middle fi nger. Since the
middle fi nger sensation is also occasionally supplied by C6
and C8, there is no conclusive way to test the C7 sensation.
Source: Hoppenfeld S. Orthopaedic Neurology. A
Diagnostic Guide to Neurologic Levels. Philadelphia,
LWW, 1997
979. Finger fl exion best tests for what nerve root? A. C5 B. C6 C. C7 D. All of the above E. None of the above
- Answer: E
Source: Wirght PD, Board Review 2004